CEN Practice Test 2 with Rationale

CEN Practice Test 2 with Rationale

Reviewing CEN Practice Questions daily can greatly improve your test readiness.

BCEN Practice Test 2 with Rationale

Question 1.
A nurse is caring for a tracheostomy patient who begins to cough and has increased secretions. The patient’s Sp02 has dropped to 89%. What is the most likely cause of this acute change?
(a) The tracheostomy tube is displaced.
(b) The patient is experiencing a pulmonary embolism.
(c) The patient has developed tracheitis.
(d) The inner cannula of the tracheostomy is obstructed.
Answer:
(d) The inner cannula of the tracheostomy is obstructed.

Increased secretions and coughing in a tracheostomy patient, along with a drop in oxygen saturation, are often signs that the inner cannula of the tracheostomy tube is obstructed. This obstruction can lead to difficulty in maintaining oxygenation and ventilation.

Question 2.    
Which of the following clinical features highly suggests a duodenal ulcer?
(a) Nocturnal pain.
(b) Hematemesis.
(c) Vomiting.
(d) Hematochezia.
Answer:
(a) Nocturnal pain.

Duodenal ulcers typically present with nocturnal pain that rouses the patient from sleep. Unlike gastric ulcers, duodenal ulcers are relieved by food, although pain can return about three hours after eating. Hematemesis and vomiting are general symptoms that are seen in both ulcers. Other symptoms of a duodenal ulcer are epigastric pain, dyspepsia, and nausea. Hematochezia, or bright red blood in stools, is not a typical symptom of duodenal ulcers.

Question 3. 
A 49-year-old male presents to the ER with complaints of severe epigastric pain of 12 hours duration. He said he has GERD and has been on his medication for five years. On examination, he is noticed to have gynecomastia. Which of the following drugs is most likely the cause of gynecomastia?    
(a) Omeprazole.
(b) Cimetidine.
(c) Aluminum hydroxide.
(d) Magnesium hydroxide.
Answer:
(b) Cimetidine.

Cimetidine is an H2 receptor blocker used in reducing stomach hyperacidity. Gynecomastia is a side effect of its long-term use due to its antiadrenergic effect.

Question 4.    
You are giving IV amphotericin B to a patient being managed for systemic histoplasmosis. An hour after commencing the infusion, the patient complains of chills and rigors. Which of the following responses is most appropriate?
(a) Switch amphotericin B to fluconazole.
(b) Administer IV prednisolone and continue with the infusion.
(c) Stop the infusion.
(d) Administer IV acetaminophen and premedicate for consequent doses.
Answer:
(d) Administer IV acetaminophen and premedicate for consequent doses.

Chills and rigors are side effects of amphotericin B. These symptoms typically subside with subsequent doses. Premedication with a steroid or an antipyretic reduces the risk of these side effects.

Question 5.    
Which of the following substrates is not expected to be present on dipstick urinalysis in a 45-year-old female with diabetic ketoacidosis?
(a) Glucose.
(b) Ketones.
(c) Protein.
(d) Bilirubin. 
Answer:
(d) Bilirubin.

Bilirubin is not expected to be present on urinalysis. Protein may be increased in patients with a suspected urinary tract infection (which, in such cases, may be a trigger for the glycemic event). Nitrates and red blood cells may also be increased.

Question 6.    
Which of the following interventions is prioritized in managing a patient with diabetic ketoacidosis?
(a) IV insulin.
(b) IV normal saline.
(c) IV potassium.
(d) IV antibiotics.
Answer:
(b) IV normal saline.

Acute volume resuscitation is the priority goal in patients with diabetic ketoacidosis. These patients are severely dehydrated, with an estimated 10 percent deficit in blood volume. Therefore, resuscitation must include volume repletion with at least 3 L of normal IV saline in the first five hours of presentation. Thereafter, other measures can be taken, like potassium repletion, insulin, and treatment of underlying infections. 

Question 7.    
A patient being managed for alcoholic ketoacidosis does not present with which of the following?
(a) Ketonemia,
(b) Hyperglycemia.
(c) Hypokalemia.
(d) Metabolic acidosis.
Answer:
(b) Hyperglycemia.

Patients with alcoholic ketoacidosis do not typically present with hyperglycemia. Those with hyperglycemia may have undiagnosed diabetes mellitus. Biochemical abnormalities in alcoholic ketoacidosis include ketonemia, metabolic acidosis, hypokalemia, hypophosphatemia, and hypomagnesemia.

Question 8.    
Which of the following is not a clinical feature of pheochromocytoma?
(a) Hypertension.
(b) Postural hypotension.
(c) Palpitations.
(d) Diarrhea.
Answer:
(d) Diarrhea.

Diarrhea is not a clinical feature of pheochromocytoma. Pheochromocytoma is characterized by an excess secretion of adrenergic hormones in the adrenal medulla. These hormones stimulate receptors that mediate the sympathetic nervous system. Therefore, clinical features of pheochromocytoma are a result of the sympathetic drive. These features include constipation, palpitations, paroxysmal hypertension, postural hypotension, tachypnea, angina, anxiety, headaches, nausea, vomiting, epigastric pain, and paresthesia.

Question 9.    
You are managing a patient with pheochromocytoma. Which of the following interventions is most appropriate to reduce the risk of paroxysmal hypertension?
(a) Place the patient in the semi-Fowler’s position.
(b) Avoid abdominal palpation.
(c) Elevate the foot of the bed.
(d) Use compression stockings.
Answer:
(b) Avoid abdominal palpation.

Paroxysmal hypertension can be triggered by palpating the tumor, which releases catecholamines. Abdominal palpations should be avoided or kept to a minimum to reduce this risk.

Question 10. 
Which of the following drugs is inappropriate for use in a 45-year-old asthmatic male being managed in the ER with malignant hypertension and ischemic cerebrovascular disease?
(a) Labetalol.
(b) Hydralazine.
(c) Nicardipine.
(d) Fenoldopam. 
Answer:
(a) Labetalol.

Labetalol is a beta-blocker with mild alpha-l-blocking activity. It causes vasodilation without causing reflex tachycardia. It is used as a bolus or a continuous infusion in patients with malignant hypertension. Its use is contraindicated in patients with asthma due taits beta-blocking activity, which can cause bronchoconstriction.

Question 11. 
Which of the following is a common side effect of procainamide?
(a) Hypotension.
(b) Polyuria.
(c) Tremors.
(d) Diaphoresis.
Answer:
(a) Hypotension.

Procainamide is a sodium channel blocker and a class 1 antiarrhythmic drug. Side effects of procainamide use are bradycardia, shock, and hypotension. Patients may also experience ventricular dysrhythmias, drug-induced lupus-like diseases, and other allergic responses.

Question 12. 
An 89-year-old female who presents to the ER with palpitations is being managed for atrial fibrillation. She is placed on a continuous infusion of amiodarone. Which of the following is a significant side effect of this drug?
(a) Colorblindness.
(b) Hepatitis.
(c) Acute kidney injury.
(d) Acute diarrhea.
Answer:
(b) Hepatitis.

Significant side effects of amiodarone are pulmonary fibrosis, thyroid dysfunction, and hepatotoxicity. It is important to monitor pulmonary, hepatic, and thyroid function while using this drug. 

Question 13.
A 58-year-oJd male who is very athletic presents to the ER with complaints of nocturnal chest pain. A history of exercise-induced angina is obtained. He is not a known hypertensive. Which of the following drugs is useful in managing this patient’s condition?
(a) Nitroglycerin.
(b) Aspirin.
(c) Esmolol.
(d) Hydralazine.
Answer:
(a) Nitroglycerin.

This patient has exercise-induced angina, which can be managed with sublingual nitrates. These drugs are potent vasodilators of arteries, veins, and arterioles. They reduce preload and afterload without reducing cardiac output and heart rate.

Question 14. 
Which of the following best describes the pathophysiology of carbon monoxide toxicity?
(a) Alveolar edema.
(b) Displacement of oxygen from hemoglobin.
(c) Depression of the respiratory center.
(d) Paralysis of respiration muscles.
Answer:
(b) Displacement of oxygen from hemoglobin.

Hemoglobin in red blood cells has a higher affinity for carbon monoxide than oxygen. In carbon monoxide poisoning, there is a displacement of oxygen and a shift of the oxygen dissociation curve to the left. Other pathophysiologic processes include inhibition of mitochondrial respiration and toxic effects of carbon monoxide on brain tissues.

Question 15.
A child who presents to the ER with ingestion of a large number of iron tablets will be managed with which of the following?
(a) Succimer.
(b) Mucomyst.
(c) Deferoxamine.
(d) Dimercaprol. 
Answer:
(c) Deferoxamine.

Deferoxamine is a chelating agent administered intravenously to bind and remove excess iron from the blood. It is commonly used in the treatment of iron poisoning. The antidote is given as an IV infusion, but it should not be administered with the goal of dropping the patient’s blodd pressure. Instead, it is used to bind to the excess iron and facilitate its elimination. Patients who are given this drug are managed with IV fluids to improve circulation.

Question 16. 
Which of the following is not a common source of lead poisoning in children?
(a) Toys.
(b) Traditional remedies.
(c) Contaminated water.
(d) Fumes from leaded gasoline.
Answer:
(d) Fumes from leaded gasoline.    

This is an unlikely source of lead poisoning in children in the United States. Common sources of poisoning include toys, jewelry, contaminated water and soil, lead-based paint chips and dust, lead-glazed pottery and ceramics, traditional home remedies, and imported candies.

Question 17. 
A person who ingests bitter almond oil is likely to be poisoned with which of the following?
(a) Methanol.
(b) Cyanide.
(c) Iron.
(d) Copper.
Answer:
(b) Cyanide.

Bitter almond oil contains hydrocyanic acid, a form of cyanide. Symptoms include tachycardia, hypotension, dizziness, syncope, seizures, acidosis, coma, and death. Treatment is with supportive measures and a cyanide kit (including hydroxocobalamin).

Question 18. 
Which of the following plants is a source of cardiac glycoside poisoning?
(a) Belladbnna.
(b) Oleander.
(c) Poison ivy.
(d) Cannabis.
Answer:
(b) Oleander.

Common oleander and yellow oleander are sources of cardiac glycoside poisoning. Other sources are woolly foxglove, purple foxglove, and lily of the valley. Belladonna is a source of atropine poisoning. Poison ivy induces type III hypersensitivity reaction. Cannabis is a hallucinogenic drug.

Question 19. 
A patient with active tuberculosis (TB) is prescribed Streptomycin as part of the treatment regimen. Which of the following side effects is the patient most at risk of developing with Streptomycin therapy?
(a) Hepatotoxicity.
(b) Ototoxicity.
(c) Nephrotoxicity.
(d) Cardiotoxicity.
Answer:
(b) Ototoxicity.

Streptomycin is an aminoglycoside antibiotic used in the treatment of tuberculosis. One of the primary side effects of aminoglycosides, including Streptomycin, is ototoxicity. This means they can cause damage to the inner ear, leading to hearing loss or balance problems. It is important to monitor patients on Streptomycin for signs of hearing loss or vertigo.

Question 20. 
A 28-year-old female in a two-month intensive therapy for pulmonary tuberculosis managed with rifampicin, isoniazid, pyrazinamide, and ethambutol is prescribed supplemental pyridoxine to reduce the risk of which of the following?
(a) Pernicious anemia.
(b) Megaloblastic anemia.
(c) Paresthesia.
(d) Optic neuritis. 
Answer:
(c) Paresthesia.

Peripheral neuropathy is a complication of isoniazid. This is because isoniazid causes pyridoxine deficiency. Peripheral neuropathy manifests as paresthesia, burning, stinging, and numbness of the hands and feet. The risk for peripheral neuropathy increases in patients with HIV, diabetes mellitus, alcoholism, cancer, and uremia. Pregnant women and elderly patients are also at risk. Pyridoxine supplement is dosed at 25 to 50 mg daily.

Question 21.
A 45-year-old homeless male presents to the ER with a history of fever, shortness of breath of three days duration, and a history of a dry, nonproductive cough of three weeks duration. Chest X-ray findings include diffuse perihilar infiltrates on both lungs. Serology reveals antibodies to HIV 1. Which of the following organisms is most implicated in his respiratory symptoms?
(a) Histoplasma capsulatum.
(b) Pneumocystis jirovecii.
(c) Mycobacterium tuberculosis.
(d) Staphylococcus aureus.
Answer:
(b) Pneumocystis jirovecii.

P. jirovecii is a ubiquitous yeast infection implicated in pneumonia in immunocompromised patients (patients with advanced HIV, patients on a systemic corticosteroid, patients with hematologic cancers, and recipients of organ transplants). This infection id suspected in immunodeficient patients with chronic dry cough. It is confirmed with histopathology.

Question 22. 
A 45-year-old male with pneumonia caused by P. jirovecii will be managed with which of the following antibiotics?
(a) Fluconazole.
(b) Trimethoprim/sulfamethoxazole.
(c) Pyrimethamine.
(d) Pyrazinamide.
Answer:
(b) Trimethoprim/sulfamethoxazole.

The first-line antibiotic for treating P. jirovecii is trimethoprim/sulfamethoxaz'ole. Second- line drugs are pentamidine, trimethoprim/dapsone, atovaquone, and clindamycin/primaquine.

Question 23. 
Which of the following measures is ineffective in reducing the risk of hospital-acquired
pneumonia in an emergency setting?
(a) Early weaning off of a ventilator.
(b) Ambulation.
(c) Prophylactic antibiotics.
(d) Isolation.
Answer:
(c) Prophylactic antibiotics.

Antibiotics should not be used as prophylaxis to prevent hospital-acquired pneumonia. This is necessary to reduce the rising prevalence of bacterial resistance to antibiotics and MRSA. Useful prevention methods are early weaning from ventilators, early ambulation and chest physiotherapy for bedridden patients, and compliance with universal and standard precautions.

Question 24. 
A 21-year-old female is being managed in the ER with sumatriptan for migraines. Which of the following best describes this drug’s mechanism of action?
(a) 5-HT receptor agonist.
(b) Vasodilator.
(c) Calcium channel blocker.
(d) Stimulates the release of neuropeptides. 
Answer:
(a) 5-HT receptor agonist.

Sumatriptan is a 5-HT receptor agonist used in treating migraines. Its mechanisms of action include vasoconstriction of meningeal arteries, inhibition of the vasoactive hormone neuropeptide, and inhibition of neurotransmission of pain within the trigeminal nerve.

Question 25.
A 45-year-old male presents to the ER and complains of headaches that he has been experiencing for the past five days. The patient describes the headaches as excruciating attacks that rapidly peak and subside quickly in an hour. The headaches are on the left side of his head and radiate in an orbitotemporal fashion. They are so severe that the patient paces back and forth until the episode ends. Which of the following diagnoses is most appropriate?
(a) Tension headaches.
(b) Cluster headaches.
(c) Migraines.
(d) Trigeminal neuralgia.
Answer:
(b) Cluster headaches.

In cluster headaches, the patient experiences excruciating unilateral pain that spreads in an orbitotemporal fashion. The headaches rapidly peak and subside spontaneously in an hour. Unlike migraines, patients who suffer from cluster headaches are restless, agitated, and unable to derive relief from sleep. The patient also experiences autonomic features like rhinorrhea, nasal congestion, and facial flushing.

Question 26. 
A 56-year-old female who presents to the ER with diplopia is likely to have a palsy in any of these cranial nerves except:
(a) II.
(b) III.
(c) IV. 
(d) VI.
Answer:
(a) II.

Cranial nerve palsy is a common cause of diplopia, which is also known as double vision. Affected nerves include the oculomotor nerve (III), which innervates most of the eye muscles; the trochlear nerve (IV), which innervates the superior oblique muscle; and the abducens nerve (VI), which innervates the lateral rectus muscle. The oculomotor nerve (III) is also responsible for accommodation.

Question 27. 
A patient who presents to the ER with headaches, diplopia, and bilateral papilledema secondary to raised intracranial pressure is being managed for pseudotumpr cerebri. Which of the following best describes the pathophysiology of raised ICP in this patient?
(a) Hydrocephalus.
(b) Cerebral tumor.
(c) Obstructed venous drainage.
(d) Meningitis.
Answer:
(c) Obstructed venous drainage.

Pseudotumor cerebri is also known as idiopathic intracranial hypertension. In this disease, the reason for the raised intracranial pressure is unknown, although obstructed venous drainage is a likely cause. This disorder is suspected in patients with features of raised ICP without cerebral tumor, malignant hypertension, hydrocephalus, or other causes of mass effect.

Question 28. 
Which of the following best describes the mechanism of action of acetazolamide in the management of idiopathic intracranial hypertension?
(a) Osmotic diuretic.
(b) Carbonic anhydrase inhibitor.
(c) Na-K-Cl transporter inhibitor.
(d) Antimineralocorticoid. 
Answer:
(b) Carbonic anhydrase inhibitor.

Acetazolamide inhibits the action of carbonic anhydrase, an enzyme involved in the reversible reaction converting carbon dioxide and water into carbonic acid. This inhibition results in decreased reabsorption of sodium and bicarbonate in the proximal renal tubule, leading to increased excretion of bicarbonate, sodium, and water in the urine. Acetazolamide is used to manage raised intracranial pressure, raised intraocular pressure, and pulmonary edema.

Question 29. 
A mother brings her 4-year-old son to the ER because she is concerned about a rash that began on his chest and has now spread to his limbs. The nurse observes that the rash consists of vesicles, pustules, and crusted spots. The child also has a low-grade fever. Which of the following is the most likely diagnosis?
(a) Measles (Rubeola).
(b) Chickenpox (Varicella).
(c) Scarlet fever.
(d) Hand, foot, and mouth disease.
Answer:
(b) Chickenpox (Varicella).

Chickenpox, or Varicella, typically starts as a rash on the chest and face and then spreads to other areas of the body. It is characterized by its distinctive progression of rash stages. Vesicles, pustules, and crusts often present simultaneously on the body. The presence of a low-grade fever also supports this diagnosis.

Question 30. 
Which of the following is not a major criterion for diagnosing infective endocarditis?
(a) Two positive blood cultures of Staphylococcus aureus.
(b) Two positive blood cultures of Coxiella burnetii.
(c) Oscillating intracardiac mass.
(d) New onset of valvular regurgitation.
Answer:
(b) Two positive blood cultures of Coxiella burnetii.

This is not a major criterion for diagnosing infective endocarditis. The criterion is one positive blood culture for Coxiella burnetii. The major criteria for diagnosing infective endocarditis are:

  • Two positive blood cultures of organisms typically implicated in infective endocarditis.
  • Serologic evidence of Coxiella burnetii presents as markedly elevated IgG antibodies.
  • One positive blood culture of Coxiella burnetii.
  • Echocardiographic findings, which are cardiac abscess, new onset of valvular regurgitation or dehiscence of prosthetic valves, and the presence of an oscillating mass on the heart valves.

Question 31. 
A 25-year-old male presented to the ER with a history of fever, weight loss, and malaise. Significant findings on examination are Osier nodes, Janeway lesions, and splinter hemorrhages. A history of IV drug abuse was obtained. Which of the following is not required to diagnose infective endocarditis according to Duke’s criteria?
(a) Two major criteria.
(b) One major and two minor criteria.
(c) One major and three minor criteria.
(d) Five minor criteria.
Answer:
(b) One major and two minor criteria.

The requirements for confirming a diagnosis of infective endocarditis are two major criteria, one major and three minor criteria, or five minor criteria. A differential diagnosis of infective endocarditis is made with one major and two minor criteria or three minor criteria.

Question 32. 
Which of the following is not a risk factor for developing endocarditis?
(a) Prosthetic heart valves.
(b) Congenital heart disease.
(c) Sepsis.
(d) Codeine addiction.
Answer:
(d) Codeine, addiction.

The risk factors for endocarditis can be divided into endocardial risk factors and bacteremia. Endocardial risk factors include rheumatic valvular disease, congenital heart disease, hypertrophic cardiomyopathy, prosthetic valves, and other intracardiac devices. The risk factors for bacteremia are IV drug abuse, sepsis, invasive dental procedures, and others.

Question 33. 
A 35-year-old male who presents to the ER with fever, pallor, and malaise is being managed for infective endocarditis. Which of the following statements is correct about antibiotic treatment in this patient?
(a) Antibiotics must be commenced before blood culture samples are collected.
(b) Standard antibiotic therapy is given for two to four weeks.
(c) Antibiotic therapy must cover gram-negative bacteria, gram-positive bacteria, and fungi.
(d) Empiric therapy must be according to local patterns of infection. 
Answer:
(d) Empiric therapy must be according to local patterns of infection.

Option A is false because antibiotics are withheld until at least two blood samples are obtained from different body sites. Thereafter, empiric antibiotic therapy can be commenced. Option B is incorrect because standard therapy lasts about six to eight weeks. Option C is incorrect because empiric therapy must cover gram-positive and gram-negative organisms. Fungi infections are not usually implicated unless the patient has severe, immuno suppression.

Question 34.
A 35-year-old male being managed for infective endocarditis is scheduled to receive IV vancomycin as part of empiric antibiotic therapy. As the nurse injects the drug as an IV bolus, she notices that the injection site is flushed and erythematous. Which of the following responses is most appropriate?
(a) Discard the drug and commence a different antibiotic.
(b) Continue with the drug administration.
(c) Inform the attending physician.
(d) Give the drug as an IV infusion.
Answer:
(d) Give the drug as an IV infusion.

This patient is experiencing red man syndrome, an adverse effect of vancomycin injection. This effect is more likely when the injection is given as a bolus injection. Interventions include flushing the line with IV normal saline and giving the drug as an IV infusion over 30 minutes.

Question 35.
A 17-year-old female who was admitted to the ER with an acute asthmatic attack is to be discharged 36 hours later. Part of her discharge requirements is oral prednisolone. Which of the following best describes the use of prednisolone in this patient?
(a) Bronchodilator.
(b) Mucolytic.
(c) Anti-inflammatory.
(d) Pneumonia prophylaxis.
Answer:
(c) Anti-inflammatory.

Prednisolone is a corticosteroid used in the treatment of asthma. As a corticosteroid, prednisolone reduces airway inflammation by suppressing the release of inflammatory mediators like leukotriene, histamine, and cytokine. Corticosteroids are used as a prophylaxis to reduce the frequency of asthmatic attacks.

Question 36. 
A two-year-old male presents to the ER with a history of cough, fever, and body weakness that he has been experiencing over the past three days. On examination, there is tachycardia, tachypnea, and hyperpyrexia. Breath sounds are vesicular, and chest expansion is equal on both sides. Percussion sounds are resonant in both hemithoraces. Chest X-ray shows patchy opacities on both lung fields. Which of the following organisms is most implicated in this patient’s condition?
(a) Mycobacterium.
(b) Candida.
(c) Respiratory syncytial virus.
(d) Haemophilus influenzae.
Answer:
(c) Respiratory syncytial virus.

Based on the history, examination, and investigation, this patient likely has bronchopneumonia. Implicated organisms in bronchopneumonia are viruses that are not as virulent as bacteria seen in lobar pneumonia. Because these organisms are not virulent, the inflammatory response is not as intense. Hence, typical chest findings on examination and X-ray are not seen.

Question 37.
Which of the following is not a clinical finding expected to be seen in a 45-year-old male admitted to the hospital with community-acquired pneumonia?
(a) Contralateral tracheal deviation.
(b) Bronchial breath sounds.
(c) Hyporesonant percussion notes.
(d) Diminished chest expansion. 
Answer:
(a) Contralateral tracheal deviation.

Expected chest findings in pneumonia include hypo resonant percussion notes from consolidation or pleural effusion, diminished chest expansion, and bronchial breath sounds. Contralateral tracheal deviation, which is the deviation of the trachea away from the site of pathology, is typically associated with large pleural effusions or tension pneumothorax and not commonly with pneumonia.

Question 38.
A 35-year-old patient is admitted to the hospital following a generalized tonic-clonic seizure. The patient’s spouse reports that the patient has been on valproate for seizure management. Which of the following laboratory tests is most important if the nurse is trying to assess for potential complications that may be related to valproate?
(a) Complete blood count (CBC).
(b) Liver function tests (LFTs).
(c) Serum electrolytes.
(d) Blood urea nitrogen (BUN) and creatinine.
Answer:
(b) Liver function tests (LFTs).

Valproate (valproic acid) is an anticonvulsant used to manage seizures, bipolar disorder, and migraines. One of the potential side effects of valproate is hepatotoxicity, which can cause liver failure. Regular monitoring of liver function tests (including AST, ALT, ALP, and bilirubin) is recommended for patients on valproate therapy so that any liver damage can be detected early.

Question 39. 
A 35-year-old male was admitted to the ER with a history of chronic cough, hemoptysis, weight loss, fever, and night sweats of three weeks duration. Chest X-ray revealed multiple cavities at the perihilar regions, with pleural effusion in the right hemothorax. Tuberculin skin tests revealed an induration that is less than 5 mm 48 hours later. The nurse will interpret the skin test as:
(a) Positive.
(b) Negative
(c) False-positive.
(d) False-negative.
Answer:
(d) False-negative.

Based on the history and chest X-ray findings, this patient has pulmonary tuberculosis. A tuberculin skin test should be positive in this patient, with an induration of more than 5 mm (usually about 10 mm). False-negative results can be seen in immunosuppressed patients, patients with hyperpyrexia, elderly patients, and patients with HIV whose CD4 count is less than 200 cells/uL.

Question 40.
Which of the following is not a clinical manifestation of dyslipidemia?
(a) Arcus cornealis.
(b) Xanthelasma.
(c) Pruritus.
(d) Paresthesia.
Answer:
(c) Pruritus.

Pruritus is not a clinical manifestation of dyslipidemia. It is a clinical feature of conjugated hyperbilirubinemia. Clinical features of dyslipidemia are arcus Cornelis, which is lipid deposits in the cornea; xanthelasma, which is deposits of cholesterol in the eyelids; tuberous and tendinous xanthomas; paresthesia; lipemia retinalis and others. 

Question 41.
An emergency nurse is reviewing the medication list of a female patient of childbearing age who is on valproate for bipolar disorder. Which of the following considerations should the nurse prioritize in the patient’s care plan?
(a) Encourage the use of oral contraceptives for birth control.
(b) Advise the patient about the risk of osteoporosis that comes with prolonged use of valproate.
(c) Discuss the potential teratogenic effects of valproate.
(d) Monitor for signs of urinary retention.
Answer:
(c) Discuss the potential teratogenic effects of valproate.

Valproate is classified as a teratogen and can cause birth defects if it is taken during pregnancy. It is important for healthcare providers to discuss the potential risks associated with valproate in women of childbearing age.

Question 42.
The living will is legally backed by which of the following?
(a) State nursing board.
(b) State law.
(c) Scope of practice.
(d) Federal law. 
Answer:
(b) State law.

A living will must conform to the standards set by state law to be valid. This means that state law sets the standard of how it should be written, signed, and witnessed. This standard varies from state to state. State laws govern the requirements and validity of a living will, but they do not directly confirm the medical condition of patients or their decision-making capacity.

Question 43.
A 68-year-old patient arrives at the ER complaining of fever, muscle aches, and confusion that began suddenly over the past few days. The patient’s spouse mentions they had recently consumed homemade soft cheese. The nurse should suspect an infection with which of the following pathogens?
(a) Escherichia coli.
(b) Salmonella.
(c) Listeria monocytogenes.
(d) Staphylococcus aureus.
Answer:
(c) Listeria monocytogenes.

Listeria monocytogenes is a bacterium that can cause a severe infection known as listeriosis. Comtnon symptoms include fever, muscle aches, and sometimes gastrointestinal symptoms. If the infection spreads to the nervous system, symptoms like confusion, loss of balance, and convulsions can occur. Ingesting contaminated food, especially unpasteurized milk and soft cheeses, can lead to this infection.

Question 44.
A 55-year-old male patient in the emergency department exhibits signs of difficulty breathing, distended neck veins, and hypotension. Which of the following conditions related to increased intrathoracic pressure is most likely the cause of these symptoms?
(a) Pneumonia.
(b) Tension pneumothorax.
(c) Pleural effusion.
(d) Atelectasis.
Answer:
(b) Tension pneumothorax.

A tension pneumothorax occurs when air continues to enter the pleural space and cannot escape, leading to increased intrathoracic pressure. This pressure compresses the lungs and other intrathoracic structures, potentially causing a shift of the mediastinum to the opposite side. The classic presentation includes difficulty breathing, distended neck veins (due to compression of the superior vena cava), and hypotension. Immediate intervention is crucial to relieve the pressure and treat the underlying cause.

Question 45. 
A patient has a crush injury after a motor vehicle accident. They may have an acute kidney injury due to:
(a) Excessive production of antidiuretic hormone (ADH).
(b) Dilutional hyponatremia.
(c) Release of myoglobin from damaged muscle tissue.
(d) Hypovolemic shock that causes decreased renal perfusion.
Answer:
(c) Release of myoglobin from damaged muscle tissue.

When a crush injury or severe muscle trauma occurs, damaged muscle cells release myoglobin into the bloodstream. When large quantities of myoglobin are filtered by the kidneys, this could indicate renal tubular obstruction and toxicity. The end result would be an acute kidney injury (AKI).

Question 46. 
Which of the following is not a feature of decorticate posturing?
(a) Flexion of the arm.
(b) Extension of the leg.
(c) Inversion of the feet.
(d) Extension of the elbows.
Answer:
(d) Extension of the elbows.

Flexion of the elbows is the main feature of decorticate posturing. In this case, the elbows are bent toward the chest, hands are tightened into fists, legs are extended, and feet are inverted. Extended elbows are seen in decerebrate posturing but not in decorticate posturing.

Question 47. 
A patient brought into the ER with decerebrate posturing has an ongoing pathology in which of the following areas?
(a) Thalamus.
(b) Cerebral hemispheres.
(c) Red nucleus.
(d) Brainstem. 
Answer:
(d) Brain stem.

Decerebrate posturing is a sign of brain stem damage, especially damage below the red nucleus. Decorticate posturing is a sign of damage to the cerebral hemispheres, thalamus, red nucleus, or internal capsule.

Question 48.
A patient who presents to the ER with acute closure glaucoma is being managed with latanoprost. Which of the following best describes this drug’s mechanism of action?
(a) Decreases pupil size.
(b) Decreases production of aqueous humor.
(c) Increases the uveoscleral outflow of aqueous humor.
(d) Causes mydriasis.
Answer:
(c) Increases the uveoscleral outflow of aqueous humor.

Latanoprost is a prostaglandin analog used in the treatment of glaucoma. It increases the uveoscleral outflow of aqueous humor, thereby reducing intraocular pressure. It does not primarily act by increasing the permeability of the sclera. Miotics like carbachol and pilocarpine are used to decrease pupillary size. Beta-blockers like timolol and esmolol and carbonic anhydrase inhibitors like acetazolamide and methazolamide reduce the secretion of aqueous humor.

Question 49.
Before dilating the eyes for ophthalmoscopy, which of the following must be excluded?
(a) Retinal detachment.
(b) Raised intraocular pressure.
(c) Corneal abrasion.
(d) Papilledema.
Answer:
(b) Raised intraocular pressure.

Before ophthalmoscopy is performed, the eyes are dilated for a proper view of the fundus of the eye. However, mydriasis can precipitate angle-closure glaucoma if the eye’s anterior chamber is shallow. Therefore, the depth of the anterior chamber should be measured with a slit lamp. Other contraindications to dilution are head injury and globe rupture.

Question 50.
A nurse in the emergency department is assessing a patient with a head injury. The nurse uses an ophthalmoscope to view the fundus of the eye. Which of the following findings would be most concerning for increased intracranial pressure?
(a) Clear, sharp disc margins.
(b) Papilledema.
(c) Constricted blood vessels.
(d) Presence of floaters.
Answer:
(b) Papilledema.

Papilledema refers to swelling of the optic disc, which is the point where the optic nerve enters the back of the eye. This swelling is often caused by increased intracranial pressure. In the context of a head injury, observing papilledema when examining the fundus of the eye would be highly concerning and suggest the possibility of increased intracranial pressure.

Question 51.
The meniscus sign on an X-ray is suggestive of which condition?
(a) Bone fracture.
(b) Pneumothorax.
(c) Pleural effusion.
(d) Pulmonary embolism.
Answer:
(c) Pleural effusion.

The meniscus sign refers to the concave appearance seen at the lung margin on an upright chest X-ray when there is a pleural effusion. It suggests fluid accumulation in the pleural space, typically due to conditions such as heart failure, pneumonia, or malignancy.

Question 52. 
A patient is being managed with topical prednisolone for allergic conjunctivitis. Long¬term use of this drug puts the patient at risk for all of the following except:
(a) Corneal ulceration.
(b) Cataracts.
(c) Glaucoma.
(d) Ectropion.
Answer:
(d) Ectropion.

Ectropion is an eversion of the eyelids. A common cause of this disorder is senility. Long-term use of topical corticosteroids places the patient at risk for ocular herpes simplex infection and consequent corneal ulceration. It also increases the risk of cataracts and glaucoma. 

Question 53. 
Which of the following is not a feature of upper motor neuron lesions?
(a) Muscle atrophy.
(b) Hyperactive reflexes.    
(c) Absent muscle fasciculations.
(d) Positive Babinski’s sign. 
Answer:
(a) Muscle atrophy.

This is a feature of lower motor neuron lesions. Features of upper motor neuron lesions are muscle spasticity, hyperactive reflexes, absent muscle fasciculations, and a positive Babinski’s sign.

Question 54. 
Which of the following is given to reduce the risk of contrast nephropathy in a patient scheduled to have a CT angiography?
(a) IV normal saline.
(b) IV allopurinol.
(c) IV mesna.
(d) IV acetylcysteine.
Answer:
(a) IV normal saline.

IV normal saline is given 6 to 12 hours before injection of contrast and then continued 6 to 12 hours after administration. Sodium bicarbonate and acetylcysteine have no proven benefits over normal saline.

Question 55. 
A patient being managed for inflammatory bowel disease is being treated with sulfasalazine. The patient will need which of the following micronutrients?
(a) Folic acid.
(b) Nicotinic acid.
(c) Ascorbic acid.
(d) Cobalamin.
Answer:
(a) Folic acid.

Sulfasalazine is a 5-aminosalicylic acid. It is an immunosuppressant that inhibits prostaglandin synthesis, leukotrienes, and other inflammatory mediators. The sulfa component of the drug inhibits the absorption of folic acid (vitamin B9). Hemolytic anemia is an adverse effect of using this drug. To reduce the risk of this occurrence, patients must take oral folic acid supplements and measure their complete blood counts every six months.

Question 56.
You are to discharge a 15-year-old male who had an emergency laparotomy for intussusception. Which of the following is not a preventive method in reducing this patient’s recurrence risk?
(a) Handwashing.
(b) Food hygiene.
(c) Rotavirus vaccination.
(d) Environmental sanitation.
Answer:
(c) Rotavirus vaccination.

Acute watery diarrhea is implicated in the risk of developing intussusceptum. This is because the intussusception rates peak when viral enteritis cases increase. However, a previous history of intussusception is a contraindication in taking the rotavirus vaccine because the patient has a higher risk of recurrence.

Question 57.
The stools of patients with intussusception are often described as:
(a) Fatty and bulky.
(b) Tarry.
(c) Currant jelly.
(d) Pale.
Answer:
(c) Currant jelly.

Patients with intussusception often present with currant jelly-like stools in the late stage. This term describes stools covered with blood and mucus resulting from rectal bleeding. Fatty and bulky stools are seen in patients with malabsorption syndromes. Tarry stools indicate upper GI bleeding, while pale stools are seen in patients with obstructive jaundice.

Question 58.
A 34-year-old female being managed for acute pancreatitis secondary to alcohol intoxication presents to the ER with Grey Turner’s sign. Which of the following best describes this sign?
(a) Tenderness on palpation of the left hypochondrium.
(b) Ecchymoses of the umbilicus.
(c) Tenderness on flexion of the back.
(d) Ecchymoses of the flanks. 
Answer:
(d) Ecchymoses of the flanks.

Grey Turner’s sign describes ecchymosis of the flanks due to retroperitoneal hemorrhage in the abdominal cavity. Ecchymosis appears as a bluish or purplish diffuse discoloration. Cullen’s sign, which is also seen in acute pancreatitis, is ecchymosis around the umbilicus. It is caused by the downward gravitation of retroperitoneal hemorrhage. 

Question 59. 
A 22-year-old male collapsed in a sauna and was brought to the hospital. He complains of severe muscle pain and weakness. His urine output has decreased, and the urine has a dark, tea-like appearance. Which of the following is the most likely diagnosis?
(a) Polymyositis.
(b) Muscular dystrophy.
(c) Rhabdomyolysis.
(d) Myasthenia gravis.
Answer:
(c) Rhabdomyolysis.

Rhabdomyolysis is a serious condition caused by the breakdown of damaged skeletal muscle tissue. It causes the release of muscle cell contents, including myoglobin, into the bloodstream. Myoglobin can cause kidney damage and reduce urine output. The dark, tea-like color of the urine is a classic sign of myoglobinuria, which is often seen in patients with rhabdomyolysis. The patient’s presentation and history of prolonged exposure in the sauna, which may have caused excessive muscle breakdown, are consistent with rhabdomyolysis.

Question 60. 
A 42-year-old female presents to the emergency department with complaints of diarrhea, abdominal cramping, and unintentional weight loss over the past two months. During the examination, the nurse documents hypoactive bowel sounds. This clinical finding is most commonly associated with which of the following?
(a) Gastrointestinal infections.
(b) Malabsorption syndromes.
(c) Irritable bowel syndrome.
(d) Acute gastroenteritis.
Answer:
(b) Malabsorption syndromes.

Malabsorption syndromes, such as celiac disease or Crohn’s disease, are conditions in which the intestines cannot absorb enough of certain nutrients and fluids. These syndromes can lead to hypoactive bowel sounds due to decreased motility and can be associated with symptoms such as diarrhea, abdominal cramping, and weight loss.

Question 61. 
A 56-year-old male presented to the ER with headache, difficulty breathing, nausea, and seizures following exposure to carbon monoxide. Which of the following best explains the toxic effect of carbon monoxide?    
(a) Shifting of the oxygen hemoglobin curve to the right.
(b) Inhibition of mitochondrial respiration.
(c) Increased alveolar dead space.
(d) Displacement of carbon dioxide from hemoglobin.
Answer:
(b) Inhibition of mitochondrial respiration.

The mechanisms of action of carbon monoxide poisoning include oxygen displacement from hemoglobin due to carbon monoxide’s high affinity for hemoglobin, inhibition of mitochondrial respiration, inhibition of the respiratory drive due to toxic effects on the brain, and shifting of the oxygen-hemoglobin dissociation curve to the left.

Question 62.
A 56-year-old male presented to the ER with a history of nonproductive cough and fatigue of three months duration and difficulty breathing of two days duration. He is a foreman who works in a remodeling and renovation company. Which of the following is most likely a risk factor in this patient?
(a) Carbon fiber.
(b) Asbestos.
(c) Mold.
(d) Chlorine gas. 
Answer:
(b) Asbestos.

Asbestos is a group of heat-resistant natural silicates used in construction, building,- automobile, and textile manufacturing. Chronic exposure to asbestos causes asbestosis, a form of pulmonary fibrosis. Risk of exposure increases in home remodelers, miners, shipbuilders, construction workers, those living close to mines, and families of workers exposed to asbestos.

Question 63. 
While inserting a chest tube for drainage of pleural effusion, it is important to avoid the neurovascular bundle by doing which of the following?
(a) Inserting the needle above the upper edge of the rib.
(b) Inserting the needle below the rib.
(c) Identifying the sternal notch.
(d) Infiltrating the area with lidocaine.
Answer:
(a) Inserting the needle above the upper edge of the rib.

Neurovascular bundles of nerves, arteries, and veins are arranged just underneath the lower edge of the rib. To avoid piercing this bundle, the needle should be inserted above the rib. To do this, the person performing the procedure must locate the first rib and palpate the intercostal spaces.

Question 64. 
Which of the following is not a characteristic of an exudate?
(a) Protein >3 g/dL.
(b) Lactate dehydrogenase <200 IU.
(c) Specific gravity >1.015.
(d) Cell count >1,000/uL.
Answer:
(b) Lactate dehydrogenase <200 IU.

This is not a characteristic of exudate. The characteristics of exudates are protein >3 g/dL, lactate dehydrogenase >200 IU, specific gravity >1.015, and cell count >1,000/uL.

Question 65.
Which of the following is not a clinical manifestation of pleural effusion?
(a) Tachypnea
(b) Tactile fremitus.
(c) Diminished chest expansion.
(d) Pleuritic chest pain.
Answer:
(b) Tactile fremitus.

Clinical manifestations of pleural effusion are tachypnea, dyspnea, absent tactile fremitus, hyporesonant percussion notes, and pleural friction rub heard on auscultation. This is a creaking or grating sound heard during inspiration and expiration.

Question 66. 
Which of the following treatments is most required in managing a patient with an Addisonian crisis?
(a) Prednisolone.
(b) Cortisol.
(c) Hydrocortisone.    
(d) Epinephrine.
Answer:
(c) Hydrocortisone.

High-dose hydrocortisone is the treatment of choice in an Addisonian crisis, too mg of hydrocortisone is given via IV over one minute. This dose is repeated every six to eight hours in the first 24 hours. Also, the patient must be resuscitated with 1 L of 5% dextrose in 0.9% saline. A delay in commencing treatment with hydrocortisone increases the risk of mortality.

Question 67. 
A 56-year-old female with thyroid storm is being managed with propranolol. This drug is required in treating all these symptoms except:
(a) Tachycardia.
(b) Tremor.
(c) Diarrhea.
(d) Exophthalmos. 
Answer:
(d) Exophthalmos.

Symptoms that do not respond to beta-blockers are goiter, weight loss, exophthalmos, increased oxygen consumption, and bruit. Propranolol is used in thyroid storms to treat the symptoms of unopposed adrenergic activity. Features that respond to beta-blocker therapy are tachycardia, heat intolerance, sweating, lid lag, proximal myopathy, and diarrhea.

Question 68.
A 28-year-old female arrives at the hospital complaining of irregular bleeding. She has never had a Pap smear. According to current guidelines, at what age should a woman begin cervical cancer screening with a Pap smear?
(a) At the age of 18.
(b) At the age of 21.
(c) Immediately after becoming sexually active.
(d) At the age of 30.
Answer:
(b) At the age of 21.

According to current guidelines, cervical cancer screening with a Pap smear should begin at 21, regardless of the level of sexual activity. Screening is recommended every three years for women aged 21-29. For women 30 and older, co-testing with a Pap smear and HPV test every five years is preferred, but a Pap smear alone every three years is acceptable.

Question 69. 
A 35-year-old female patient is concerned about her risk of cervical cancer because her mother was diagnosed with it at the age of 40. She wants to know the best screening method. Which combination of tests offers the most comprehensive screening for her age?
(a) Pap smear alone.
(b) HPV testing alone.
(c) Both Pap smear and HPV testing.
(d) Pelvic ultrasound.
Answer:
(c) Both Pap smear and HPV testing.

For women aged 30 to 65, co-testing with both Pap smear and HPV test every five years provides a more comprehensive screening than using either test alone. The Pap smear evaluates for cellular changes in the cervix that may be precancerous, and the HPV test checks for the presence of high-risk HPV strains known to be the primary cause of cervical cancer.

Question 70. 
Which of the following is not a direct complication of hyperpyrexia?
(a) DIC.
(b) End-organ failure.
(c) Seizures.
(d) Anemia.
Answer:
(d) Anemia.

Hyperpyrexia is defined as a core temperature that is greater than 4i°C (105.8 °F). Complications of hyperpyrexia include denaturation of enzymes, end-organ damage, activation of the coagulation cascade, DIC, and cardiopulmonary stress. In children, a febrile seizure is a common occurrence. 

Question 71. 
A 56-year-old female is placed on digoxin therapy for left-sided heart failure. In the case of toxicity, which of the following measures is not useful in the management of this patient?
(a) Digoxin fab.
(b) Magnesium sulfate.
(c) Lidocaine.
(d) Calcium gluconate.
Answer:
(d) Calcium gluconate.

Digoxin fab is the primary treatment of digoxin toxicity. It is an antibody that consists of fragments of anti digoxin immunoglobulin. Digoxin fab is used to treat severe hyperkalemia and arrhythmias. Other treatment modalities are magnesium sulfate, which is used to treat ventricular arrhythmias; lidocaine, a class l antiarrhythmic drug; and phenytoin.
Brady arrhythmias are treated with atropine or catecholamines.

Question 72. 
Which of the following management principles is unsuitable for a 67-year-old female with congestive heart failure?
(a) Serial weight monitoring.
(b) Anticoagulant therapy.
(c) Sodium restriction.
(d) Salt-poor albumin infusion. 
Answer:
(d) Salt-poor albumin infusion.

This treatment is inappropriate in this patient with edema caused by increased hydrostatic pressure. Salt-poor albumin infusion is used in patients with edema caused by decreased oncotic pressure (i.e., hypoproteinemia).

Question 73. 
A 50-year-old male patient has swelling and redness on his left leg after a bee sting. Assessment identifies localized edema around the site of the sting. This type of response is indicative of:
(a) An ischemic reaction.
(b) Decreased intracellular osmotic pressure.
(c) Increased capillary permeability.
(d) An allergic reaction to a substance that is unrelated to the bee sting.
Answer:
(c) Increased capillary permeability.

Increased capillary permeability allows fluids and proteins to leak out of the capillaries and into the surrounding tissues. This response is seen when inflammation occurs due to injury or allergic reactions, such as a bee sting. The edema (swelling) observed is a direct result of this fluid leakage. Although a bee sting can induce an allergic reaction, the localized swelling at the sting site is primarily due tp increased capillary permeability.

Question 74. 
A patient with STEMI secondary to atherosclerosis is being managed with rosuvastatin. Which of the following is an adverse effect of this drug?
(a) Flatulence
(b) Pruritus
(c) Elevated liver enzymes
(d) Hyperuricemia
Answer:
(c) Elevated liver enzymes.

Rosuvastatin is an HMG CoA reductase inhibitor that reduces serum concentrations of LDL. Adverse effects of this drug are rare. However, they include myositis, elevated liver enzymes, and rhabdomyolysis. Adverse effects are more likely in older patients and - patients taking multiple drugs.

Question 75. 
A nurse is’caring for a patient with severe sepsis. Which symptom would the nurse anticipate due to increased capillary permeability associated with the inflammatory response in sepsis?
(a) Polyuria.
(b) Hypertension.
(c) Pulmonary edema.
(d) Hyperkalemia.
Answer:
(c) Pulmonary edema.

In severe sepsis, systemic inflammatory responses increase capillary permeability throughout the body. This increased permeability allows fluids to leak out of the vascular space and into the interstitial and, eventually, intracellular spaces. In the lungs, this manifests as pulmonary edema, which is an accumulation of fluid in the alveoli that can lead to impaired gas exchange and respiratory failure. The other options are not directly related to increased capillary permeability.

Question 76. 
A patient who has sustained a corneal abrasion will fluoresce which of the following colors after staining?
(a) Yellow.
(b) Orange.
(c) Green.
(d) Blue.
Answer:
(c) Green.

The fluorescein stain, which is orange, appears green in corneal ulcers on slit lamp examinations. This stain is useful in highlighting the edges and margins of the ulcer. 

Question 77. 
Which of the following interventions must be prioritized in a patient who presents to the ER with a chemical injury to the eye?
(a) Irrigation with normal saline.
(b) Fluorescein staining.
(c) Visual acuity test.
(d) Tonometry. 
Answer:
(a) Irrigation with normal saline.

Chemical burns are more extensive than thermal burns. Acids denature and coagulate the proteins in the eye, thereby preventing further absorption of the acids. On the other hand, Alkalis liquefy the eye proteins and penetrate to affect other tissues, thereby causing more extensive burns. The first intervention is to irrigate the eye with copious amounts of normal saline or a borate buffer solution. Irrigation should ideally be done under ocular anesthesia. Irrigation is necessary to remove all traces of the toxic agent and prevent further ocular injury. Irrigation should continue until the pH of the eye is between 7 and 7.2.

Question 78. 
Eye burns caused by acids tend to be less destructive than their alkali counterparts. Which of the following best explains the reason for this?
(a) Acids coagulate eye proteins.
(b) Acids have a lower pH than alkalis.
(c) Acids are more easily metabolized than alkalis.
(d) Acids do not trigger a rapid immune response.
Answer:
(a) Acids coagulate eye proteins.

Acids have a less destructive action on the eye than their alkali counterparts because acids denature and coagulate the proteins in the eyes, forming an impenetrable barrier. Alkalis are more lipophilic and can penetrate the deeper tissues of the eye.

Question 79. 
A patient who presents to the ER with a corneal abrasion is being managed with homatropine. Which of the following best describes the rationale behind using this drug?
(a) Lubrication.
(b) Mydriasis.
(c) Antibiotics.
(d) Diuretic.
Answer:
(b) Mydriasis.

Homatropine is a cycloplegic drug used to induce mydriasis (pupillary dilation). As a muscarinic receptor antagonist, it inhibits the action of acetylcholine on the pupillary reflexes responsible for constriction and accommodation.

Question 80.
To measure the intraocular pressure of the eye, which of the following must first be administered? 1
(a) Bacitracin.
(b) Fluorescein.
(c) Proparacaine.    
(d) Tropicamide.
Answer:
(c) Proparacaine.

Tonometry is done to measure the intraocular pressure of the eye. It must be performed under anesthesia because the eye’s cornea is indented. To minimize pain, local anesthesia must be used.

Question 81. 
A patient who presents to the ER with acute acetaminophen poisoning is being managed with N-acetylcysteine. This drug reduces the toxic effects of acetaminophen on which of the following cells?
(a) Neurons.
(b) Hepatocytes.
(c) Nephrons.
(d) Cardiomyocytes.
Answer:
(b) Hepatocytes.

N-acetylcysteine is a precursor of glutathione. When administered, N-acetylcysteine is metabolized to glutathione. Glutathione binds to the toxic metabolite of acetaminophen before it damages the hepatocytes. This bound substrate is then excreted.

Question 82.
A four-year-old child who presents to the ER with acute salicylate poisoning is being managed with activated charcoal. Which of the following best describes the mechanism of action of this substance?
(a) Induces vomiting.
(b) Induces diuresis.
(c) Increases absorption.
(d) Increases elimination. 
Answer:
(d) Increases elimination.

Activated charcoal is used to prevent the absorption of toxic materials into the systemic circulation. Because of its large surface area, activated charcoal absorbs and binds to toxic materials, making them inert and facilitating their excretion via the feces. 

Question 83.
A 70-year-old individual comes to the hospital and reports episodes of bright red rectal bleeding over the past several weeks. After a colonoscopy, a vascular abnormality is detected in the cecum. What is the most likely diagnosis?
(a) Cecal adenocarcinoma.
(b) Cecal angiodysplasia.
(c) Appendicitis.
(d) Cecal diverticulitis.
Answer:
(b) Cecal angiodysplasia.

Angiodysplasia is a condition where small blood vessels in the wall of the intestines dilate and cause bleeding. It is most common in the elderly population. These vascular malformations are often found in the right colon, particularly in the cecum.

Question 84. 
Which of the following is not a clinical feature of tonsillar herniation?
(a) Tachycardia.
(b) Systolic hypertension.
(c) Widened pulse pressure.
(d) Abnormal breathing.
Answer:
(a) Tachycardia.

Tonsillar herniation is the herniation of the cerebellar tonsils through the foramen magnum and subsequent compression of the medulla oblongata against the odontoid process. This compression is called coning, which manifests clinically as Cushing’s triad. Cushing’s triad includes bradycardia, systolic hypertension (widened pulse pressure), and abnormal respiration. Progressive coning leads to cardiopulmonary failure and death. Tachycardia is not a clinical feature of the condition.

Question 85.
A 56-year-old male presents to the ER with a traumatic brain injury following a fall. As the emergency nurse on call, you must quickly assess all the brain stem reflexes. Which of the following is not useful?
(a) Pupillary reflex.
(b) Corneal reflex.
(c) GCS.
(d) Respiratory pattern.
Answer:
(c) GCS.

The Glasgow Coma Scale is used to assess the consciousness of a patient. Consciousness is mediated by different parts of the brain, not just the brain stem. Hence, it is not solely a test for brain stem function. Tests for brain stem function include assessment of pupillary reflex, ocular reflex, respiratory pattern, apd ocular movement.

Question 86. 
A 67-year-old male presents to the ER with traumatic brain death following a home accident. On examination, the pupils are round and mid-sized but do not respond to light. The hemorrhage is most likely located in which of the following areas?
(a) Cerebral cortex.
(b) Midbrain.
(c) Cerebellum.
(d) Pons.
Answer:
(b) Midbrain.

Lesions in the midbrain present as round, midsized pupils that do not respond to light. Pontine lesions cause pinpoint pupils; lesions in the lateral medulla often present as ipsilateral Horner’s syndrome.

Question 87. 
Which of the following describes the doll’s eye movement?
(a) Conjugate movement of the eyes in the opposite direction as the head when rotated.
(b) Conjugate movement of the eyes in the same direction as the head when rotated.
(c) Disconjugate movement of the eyes.
(d) Flickering of the eyelids.
Answer:
(a) Conjugate movement of the eyes in the opposite direction as the head when rotated. 

The doll’s eye movement indicates an intact oculocephalic reflex. In this movement, the two eyes move in the opposite direction of the head when rotated. This means that if the head is rotated to the left, the eyes move to the right and vice versa. Please note that this reflex cannot be elicited in an awake patient.

Question 88. 
Which of the following is not a lateralization sign?
(a) Anisocoria.
(b) Unilateral Babinski’s sign.
(c) Muscle flaccidity.
(d) Facial asymmetry.
Answer:
(c) Muscle flaccidity.

Muscle flaccidity is not a lateralization sign. Lateralizing signs point to a local (often structural) pathology in the brain. These signs are unequal pupil sizes (anisocoria), unilateral Babinski’s sign, facial asymmetry, unilateral focal fits, asymmetric deep reflexes, unilateral hypotonia, and movement of the eyes to one side. 

Question 89.
A 45-year-old female patient presents to the emergency department and reports a recent loss of her sense of smell after a minor head injury. This symptom is referred to as:
(a) Aphasia.
(b) Anosmia.
(c) Amnesia.
(d) Agnosia.
Answer:
(b) Anosmia.

Anosmia is the medical term for the loss of the sense of smell. It can be caused by various factors, including head trauma, infections, or conditions like Parkinson’s disease. In the context of a head injury, it is important to assess for anosmia as it can indicate an injury to the olfactory nerve.

Question 90. 
A 35-year-old male presents to the ER with a traumatic brain injury following a road traffic accident, pn examination, the patient makes incomprehensible sounds in response to the depressioh of his nail beds. However, his eyes remain closed. He also shows a flexion response to pain. What is his GCS score?
(a) 4.
(b) 5.
(c) 6.
(d) 7.
Answer:
(c) 6.

The patient has an eye response of 1, a verbal response of 2, and a motor response of 3. His total GCS score is 6.

Question 91.
Which of the following is not a source of carbon monoxide poisoning?
(a) Automobiles.
(b) Gas heaters.
(c) Kerosene heaters.
(d) Tobacco smoke.
Answer:
(d) Tobacco smoke.

Although tobacco smoke contains carbon monoxide, the amount is not enough to cause poisoning. Sources of carbon monoxide poisoning include furnaces, gas heaters, kerosene heaters, automobiles, charcoal-burning stoves, and water heaters.

Question 92.
A patient with a known history of chronic kidney disease presents with painless hematochezia. No evidence of diverticular disease or colorectal cancer is found on imaging. What could be a possible vascular etiology for this presentation?
(a) Colonic diverticulosis.
(b) Colonic adenoma.
(c) Colonic angiodysplasia.
(d) Colonic polyps. 
Answer:
(c) Colonic angiodysplasia.

Colonic angiodysplasia is an abnormality in the blood vessels of the colon, which can cause bleeding. Patients with chronic kidney disease have a higher incidence of angiodysplasia. Hematochezia, which refers to the passage of fresh blood through the rectum, is a common symptom of colonic angiodysplasia when there is active bleeding.

Question 93.
A 35-year-old woman comes to the ER complaining of heavy menstrual bleeding and pelvic pain. On examination, the uterus is enlarged and irregular. What is the most likely diagnosis?
(a) Endometriosis.
(b) Ovarian cyst.
(c) Uterine fibroids.
(d) Pelvic inflammatory disease.
Answer:
(c) Uterine fibroids.

Uterine fibroids, or leiomyomas, are benign tumors that arise from the muscle tissue df the uterus. They are common in women in their reproductive years and can cause symptoms like heavy menstrual bleeding and pelvic pain. A main sign of uterine fibroids is an enlarged and irregular uterus.

Question 94. 
A mother who is Hepatitis B surface antigen (HBsAg) positive gives birth. What is the most appropriate management action to prevent Hepatitis B transmission to the baby?
(a) Start the Hepatitis B vaccine series within 24 hours.
(b) Administer hepatitis B immune globulin (HBIG) within 12 hours of birth.
(c) Both the Hepatitis B vaccine and HBIG should be administered within 12 hours of birth.
(d) Delay the Hepatitis B vaccine series until the child is six months old.
Answer:
(c) Both the Hepatitis B vaccine and HBIG should be administered within 12 hours of birth.

To prevent perinatal transmission of Hepatitis B, the CDC recommends that all infants born to HBsAg-positive mothers receive hepatitis B immune globulin (HBIG) and the first dose of the Hepatitis B vaccine within 12 hours of birth. This provides the newborn with immediate passive immunity from HBIG and starts the process of active immunity with the vaccine. Delaying the vaccine or giving only one of the two treatments decreases the efficacy of transmission prevention. 

Question 95. 
A 22-year-old woman comes to the ER complaining of a painless, mobile lump in her left breast that she noticed a month ago. The lump is well-defined and rubbery on palpation. There are no overlying skin changes and no nipple discharge. What is the most likely diagnosis?
(a) Breast abscess.
(b) Fibroadenoma.
(c) Inflammatory breast cancer.
(d) Mastitis.
Answer:
(b) Fibroadenoma.

Fibroadenomas are benign breast tumors that are common in young women. They are typically well-defined, rubbery, mobile, and non-tender.

Question 96. 
A 22-year-old female comes to the hospital 48 hours after having unprotected intercourse. She is concerned that she might be pregnant. Which of the following is the most appropriate intervention?
(a) Advise her that it is too late to prevent pregnancy post-intercourse.
(b) Prescribe levonorgestrel to be taken as soon as possible.
(c) Start her on a regular oral contraceptive pill.
(d) Perform a pregnancy test before any intervention.
Answer:
Prescribe levonorgestrel to be taken as soon as possible.

Levonorgestrel is an emergency contraceptive that can be taken up to 72 hours after unprotected intercourse to prevent pregnancy. The sooner it is taken after the unprotected event, the more effective it is. Starting a regular oral contraceptive pill without addressing the immediate risk is not the most appropriate initial step. Although it is always good to rule out an existing pregnancy with a pregnancy test, the window for levonorgestrel’s efficacy is limited, so it is important to act promptly.

Question 97.
A Westermark sign is observed on a chest radiograph. This sign is mainly associated with which of the following conditions?
(a) Pneumothorax.
(b) Pulmonary embolism.
(c) Congestive heart failure.
(d) Tuberculosis. 
Answer:
(b) Pulmonary embolism.

The Westermark sign is a radiological sign that may be present in cases of pulmonary embolism. It represents a decrease or absence of blood flow in a localized region of the lung on a chest radiograph. Although it is not always present in pulmonary embolism and is not the most common sign, its presence can provide supportive evidence for the diagnosis. The other options are associated with different radiographic findings.

Question 98. 
A 15-year-old female with acute glomerulonephritis is being managed in the ER. Which of the following is a likely history in this patient?
(a) Recent sexual history.
(b) Recent history of sore throat.
(c) History of abdominal trauma.
(d) Family history of acute glomerulonephritis.
Answer:
(b) Recent history of sore throat.

Postinfectious glomerulonephritis caused by group A beta-hemolytic streptococcus is a common cause of glomerulonephritis. A recent history of sore throat or impetigo (about 6 to 21 days) before renal symptoms is typical.

Question 99. 
Which of the following is a typical presentation in a patient with acute glomerulonephritis?
(a) Dysuria and flank pain.
(b) Hematuria and proteinuria.
(c) Anasarca and oliguria.
(d) Ascites and hypertension.
Answer:
(b) Hematuria and proteinuria.

Clinical features of acute glomerulonephritis are hematuria, proteinuria, and hypertension. Clinical features come in a spectrum. Patients with extreme cases present with oliguria and acute kidney injury. Dysuria and flank pain are common findings in urinary tract infections. Anasarca and oliguria are findings in nephrotic syndrome, while ascites and hypertension are seen in acute kidney injury.

Question 100.    
Which of the following is not a biochemical indication for dialysis?
(a) Potassium - 6.5 mEq/L.
(b) Creatinine - 12 mg/dL.
(c) pH-7.1.
(d) Sodium - 130 mEq/L.
Answer:
(d) Sodium - 130 mEq/L.

Dialysis is not typically indicated for mild hyponatremia. Instead, the primary biochemical reasons for initiating dialysis include:

  • Severe hyperkalemia (potassium levels > 6.5 mEq/L) that doesn’t respond to other treatments.
  • Uremia that presents with symptoms like uremic encephalopathy, gastritis, and pericarditis. 
  • Serum creatinine levels > 12 mg/dL.
  • Acidosis.

Question 101. 
A 26-year-old male arrives at the hospital after falling off his bicycle. He complains of pain and tenderness over the upper chest, specifically pointing toward the right clavicle. There is a noticeable bump in that area. Which of the following is the most likely diagnosis?
(a) Rib fracture.
(b) Pneumothorax.
(c) Clavicle fracture.
(d) Sternoclavicular joint dislocation.
Answer:
(c) Clavicle fracture.

The clavicle, or collarbone, is a commonly fractured bone, especially if there is direct trauma to the shoulder. The patient’s history of a fall, localized pain, tenderness, and a noticeable bump over the clavicle region are characteristic signs of a clavicle fracture.

Question 102.    
A 56-year-old male who had hemodialysis due to acute kidney injury complains of headaches, restlessness, confusion, and vomiting. Which of the following best explains the pathophysiology behind these symptoms?
(a) Rapid removal of urea.
(b) Severe hypotension.
(c) Redistribution of fluid in the nephrons.
(d) Anxiety disorder. 
Answer:
(a) Rapid removal of urea.

This patient is experiencing disequilibrium syndrome caused by the rapid removal of urea from the blood. Urea has a small osmotic effect on the plasma. Hence, it is rapidly removed. However, the clearance rate is faster than its diffusion in the blood-brain barrier. This creates a gradient between the plasma and brain cells, leading to cerebral edema.

Question 103.
A patient being managed for severe anemia secondary to chronic kidney disease complains of pruritus. Which of the following is a likely cause?
(a) Potassium.
(b) Urea.
(c) Bilirubin.
(d) Uric acid.
Answer:
(b) Urea.

Uremia is a common cause of pruritus in patients with chronic kidney disease (CKD). This form of pruritus is worse at night and is severe enough to disrupt sleep. Areas that are often involved are the abdomen, back, arms, 'and head. Increased serum calcium and phosphorus are other causes of pruritus in patients with CKD.

Question 104.
A 54-year-old male has gone into respiratory arrest and was intubated with an endotracheal tube to commence assisted ventilation. Which of the following drugs is most suitable for blunting any potential gag reflex?
(a) Succinylcholine.
(b) Atropine.
(c) Atracurium.
(d) Mivacurium.
Answer:
(a) Succinylcholine.

Succinylcholine is a reversible neuromuscular blocker with a very rapid onset of action (30 seconds to 1 minute). This pharmacokinetic property makes it suitable for use in emergency cases to blunt a potential gag reflex. However, succinylcholine is contraindicated in patients with eye injuries, renal failure, crush injuries, spinal cord injuries, and burns. Atracurium and mivacurium are nondepolarizing neuromuscular blockers with a long onset of action and long duration.

Question 105.    
A 15-year-old male with apnea caused by severe asthma is about to be intubated. Ketamine is administered to sedate him. Which of the following side effects of ketamine should the nurse be aware of?
(a) Hypotension.
(b) Hallucinations.
(c) Malignant hyperthermia.
(d) Bradycardia.
Answer:
(b) Hallucinations.

Ketamine causes dissociative anesthesia. Side effects are hallucinations, delirium, frightening dreams, and nightmares. Because ketamine stimulates the cardiac centers, hypertension and tachycardia are also side effects. Ketamine has bronchodilation effects that are therapeutic in this patient.

Question 106.    
Which of the following drugs is not a cause of priapism?
(a) Prazosin.
(b) Sildenafil.
(c) Amphetamine.
(d) Spironolactone.
Answer:
(d) Spironolactone.

Spironolactone is not a cause of priapism. It is an aldosterone antagonist used in managing patients with hypertension, heart failure, and hypertensive heart disease. A common side 
effect of this drug is gynecomastia due to its antiadrenergic effect. Drugs that are implicated in priapism are drugs used for erectile dysfunction; alprostadil and phosphodiesterase 5 inhibitors; recreational drugs like cocaine and amphetamine; alpha-blockers like prazosin and tamsulosin; antipsychotics; antihypertensives like nifedipine, warfarin, lithium, and corticosteroids; and hypoglycemic agents like tolbutamide.

Question 107.    
Which of the following immediate interventions is most appropriate in a 35-year-old male who presents to the ER with priapism secondary to recreational drug abuse?
(a) Electrophoresis.
(b) Ice pack therapy.
(c) Penile shunt.
(d) Phenylephrine injection. 
Answer:
(d) Phenylephrine injection.

Treatment for ischemic priapism must be commenced immediately to reduce the risk of penile gangrene. Immediate treatment involves aspiration of blood to the base of the corpora cavernosa with a nonheparinized syringe and saline irrigation with an injection of phenylephrine into the cavernous. If these interventions are unsuccessful (i.e., priapism lasts more than 48 hours), a surgical shunt between the corpus cavernosum and glans penis is quickly created. Ice pack therapy is used for patients with nonischemic priapism. Plasma electrophoresis is used in patients with priapism secondary to sickle cell disease.

Question 108.    
Which of the following is an immediate intervention in a patient who presents to the ER with flank pain, fever, nausea, and vomiting due to nephrolithiasis?
(a) Morphine.
(b) Potassium citrate.
(c) Metoclopramide.
(d) Acetazolamide.
Answer:
(a) Morphine.

Analgesia such as morphine is an immediate intervention in this patient, who is in severe distress. After adequate analgesia, other measures, like antipyretics, antiemetics, and fluid administration, can be commenced. Definite treatment is the removal of the calculi either via surgery, radiology, or drugs.

Question 109.    
A 56-year-old male who presents to the ER with severe abdominal pain, nausea, and vomiting is being managed with tamsulosin. Which of the following best describes the rationale behind using this drug?
(a) Alkalinization of the urine.
(b) Analgesia.
(c) Dissolution of calculi.
(d) Excretion of calculi.
Answer:
(d) Excretion of calculi.

Tamsulosin is an alpha-receptor blocker and muscle relaxant useful in enhancing the passage of large calculi. Some studies have shown that tamsulosin is effective in enhancing the passage of ureteral stones. Its comparison to calcium channel blockers varies based on studies, with some showing similar efficacy.

Question 110.    
Which of the following features distinguishes testicular torsion from epididymitis?
(a) Testicular induration.
(b) Transillumination.
(c) Absence of cremasteric reflex.
(d) Urinary frequency.
Answer:
(c) Absence of cremasteric reflex.

The clinical features of testicular torsion and epididymitis often overlap, as they may both present with scrotal pain, fever, urinary frequency, scrotal edema, and induration. However, in testicular torsion, the cremasteric reflex on the affected testes is usually absent.

Question 111. 
Which of the following is a potential complication of clavicle fractures?
(a) Aortic rupture.
(b) Brachial plexus injury.
(c) Gastric perforation.
(d) Laryngeal edema.
Answer:
(b) Brachial plexus injury.

The brachial plexus is a network of nerves that supplies the skin and musculature of the upper limb, including the shoulder, arm, and hand. A clavicle fracture can potentially damage or impinge on these nerves, which can cause a brachial plexus injury. Symptoms might include numbness, tingling, or weakness in the arm. Although aortic rupture, gastric perforation, and laryngeal edema are serious conditions, they are not directly related to clavicle fractures.

Question 112.    
Which of the following measures is most appropriate in reducing the spread risk of viral conjunctivitis?
(a) Eye shield.
(b) Handwashing.
(c) Topical antibiotics.
(d) Cool compresses. 
Answer:
(b) Handwashing.

Handwashing is very useful in reducing the spread risk of viral conjunctivitis. Patients are counseled to wash their hands after touching their eyes and face. All health workers and caregivers in direct contact with the patient must wash their hands after visitation. Viral conjunctivitis is self-limiting, so antibiotics and antivirals are not useful. The eyes of affected patients must not be patched to reduce the risk of prolonged infection and viral replication. Cool compresses are not used to reduce the risk of spread. They are used to provide symptomatic relief.

Question 113. 
A 45-year-old female patient arrives at the hospital with severe lower abdominal pain. She rates her pain as an eight on a scale of zero to ten. Before administering analgesia, which of the following should the nurse assess?
(a) The patient’s blood glucose level.
(b) Time of the patient’s last meal.
(c) The patient’s allergy history.
(d) The patient’s body mass index (BMI).
Answer:
(c) The patient’s allergy history.

Before administering any medication, including analgesics, assessing the patient’s allergy history is vital t^ prevent allergic reactions or anaphylaxis. Although the other options may be relevant, they are not the primary concern before administering analgesics.

Question 114.    
Which of the following is the earliest symptom of compartment syndrome?
(a) Pain.
(b) Paresthesia.
(c) Paralysis.
(d) Pulselessness.
Answer:
(a) Pain.    

Worsening pain is the earliest sign of compartment syndrome. The patient often complains of pain that is greater than the degree of injury. This pain can also be elicited by passively stretching the muscles in the affected compartment. Pallor and pulselessness are late signs.

Question 115. 
A 55-year-old female who presents to the ER with diaphoresis, fast breathing, cough, and hemoptysis has just been diagnosed with interstitial fibrosis. Which of the following is not a useful finding on a chest X-ray?
(a) Meniscus sign.
(b) Kerley B lines.
(c) Unfolding of the aorta.
(d) Ground glass appearance.
Answer:
(d) Ground glass appearance.

Ground glass appearance on imaging is a non-specific finding and can be seen in various lung diseases, including certain types of pneumonia, interstitial lung diseases, and pulmonary edema. It is not specific to interstitial fibrosis. Chest X-ray features of heart failure are pleural effusion (meniscus sign); boot-shaped heart (enlarged cardiac size); Kerley B lines, which are seen in the lower lung field; unfolding of the aorta; fluid in the major fissure; and alveolar edema.

Question 116.
A 67-year-old male is being managed for left ventricular failure with preserved ejection fraction. All these drugs are beneficial to this patient except:
(a) Losartan.
(b) Spironolactone.
(c) Atenolol.
(d) Furosemide.
Answer:
(c) Atenolol.

Atenolol is a beta-blocker that reduces heart rate and the force of heart muscle contraction, leading to decreased blood pressure. The effect of beta-blockers is harmful to patients with heart failure with preserved ejection fraction. This is because patients with this disorder already have severe diastolic dysfunction. Their cardiac output is, therefore, dependent on the heart rate. Reducing their heart rate can worsen their symptoms. 

Question 117.
Which of the following is not an expected clinical finding in a 67-year-old female being managed for right-sided heart failure?
(a) Distended neck veins.
(b) Pedal edema.
(c) Tender hepatomegaly.
(d) Bilateral crepitations. 
Answer:
Bilateral crepitations.

Bilateral crepitations are a clinical finding in left-sided heart failure. In this condition, there is a backflow of blood from the left ventricle into the pulmonary veins. This causes pulmonary edema and congestion that manifests as breathlessness, chest tightness, hemoptysis, and bilateral crepitations at the basal lung fields.

Question 118.
A four-year-old male presents to the ER with a history of sore throat, drooling, fever, and irritability of three days duration. On examination, there is hyperpyrexia, tachycardia, and tachypnea. The child is also noted to stay in the tripod position. An X-ray of the neck shows a classic thumb sign. Which of the following diagnoses is most likely?
(a) Croup.
(b) Epiglottitis.
(c) Tonsillitis.
(d) Pharyngitis.
Answer:
(b) Epiglottitis.

Although the clinical features of croup and epiglottitis may overlap, the thumb sign on the neck X-ray confirms epiglottitis. Please note that acute epiglottitis requires immediate securing of the airway by qualified personnel in an operating room. This is because the risk of respiratory obstruction is very high.

Question 119. 
Which of the following side effects should a patient be monitored for after administration of morphine for pain relief?
(a) Tachycardia.
(b) Hyperactivity.
(c) Respiratory depression.
(d) Diuresis.
Answer:
(c) Respiratory depression.

Morphine is an opioid analgesic that relieves moderate to severe pain. However, one of the most serious side effects of opioids (including morphine) is respiratory depression. Nurses should monitor the patient’s respiratory rate, rhythm, and depth after administering morphine. The other options are not the primary side effects of morphine.

Question 120.    
A five-year-old male presents to the ER with cough, stridor fever, and malaise. There is a gray exudate in the tonsillar area of the throat, which is fibrinous and difficult to scrape. Scraping causes bleeding. Which of the following organisms is most implicated?
(a) Pertussis.
(b) Haemophilus.
(c) Corynebacterium.
(d) Streptococcus.
Answer:
(c) Corynebacterium.

Corynebacterium diphtheriae is implicated in infections of the nasopharynx. Clinical features include sore throat, fever, malaise, tachycardia, headaches, chills, and dysphagia. On throat examination, there is a characteristic exudate in the tonsillar area. This exudate is initially white and glossy but becomes grayish and fibrinous. It is so adherent to the underlying mucosa that scraping causes bleeding.

Question 121.    
You are performing an NIPPV on a 15-year-old male who presents to the ER with severe acute asthma. Which of the following is a sign of hyperventilation?
(a) Tachypnea.
(b) Stomach insufflation.
(c) Increased oral secretions.
(d) Increased chest expansions.
Answer:
(b) Stomach insufflation.

Evidence of hyperventilation through an endotracheal tube includes stomach insufflation, tension pneumothorax, and aspiration.

Question 122.    
A 58-year-old male patient comes to the hospital with signs of fatigue, loss of appetite, and confusion. Lab tests reveal an elevated serum urea level. This could indicate:
(a) Hypoglycemia.
(b) Renal dysfunction.
(c) A liver enzyme deficiency.
(d) Dehydration.
Answer:
(b) Renal dysfunction.

Urea is a waste product formed in the liver (an end product of protein metabolism) and eliminated in the urine. Elevated serum urea levels (often termed Blood Urea Nitrogen or BUN) are most commonly associated with renal dysfunction. This is because the kidneys are vital in filtering and excreting urea. If they are not working properly, urea can build up in the bloodstream. 

Question 123.    
A 35-year-old patient with rheumatoid arthritis has been prescribed prednisolone. Which of the following is a potential side effect that the patient should be monitored?
(a) Bradycardia.
(b) Hypotension.
(c) Hyperglycemia.
(d) Respiratory alkalosis.
Answer:
(c) Hyperglycemia.

Prednisolone is a corticosteroid used to treat various conditions by reducing inflammation. One of the known side effects of corticosteroids, including prednisolone, is hyperglycemia (elevated blood sugar levels). This can be particularly concerning in patients who are diabetic or predisposed to diabetes.

Question 124.    
A 55-year-old female patient with raised intracranial pressure is being managed in the ER. Part of her treatment includes hyperventilation with supplemental oxygen. Which of the following best describes the mechanism of action of hyperventilation on ICP?
(a) Vasoconstriction.
(b) Vasodilation.
(c) Reduced cerebral perfusion.
(d) Reduced systemic blood pressure.
Answer:
(a) Vasoconstriction.

Hyperventilation with supplemental oxygen washes out carbon dioxide and induces hypocapnia. As a result, the cerebral arteries constrict to reduce cerebral blood perfusion, cerebral blood volume, and intracranial pressure.

Question 125.    
Which of the following is the most appropriate treatment for Clostridium difficile-induced diarrhea?
(a) Clindamycin.
(b) Probiotics.
(c) Vancomycin.
(d) Metronidazole.
Answer:
(c) Vancomycin.

Oral vancomycin is given for its antibiotic properties, primarily to treat Clostridioides difficile (previously known as Clostridium difficile) infections in the colon. Since it is not absorbed into systemic circulation, it remains in the gut and exerts its antimicrobial properties on the gut mucosa. Probiotics have not been proven useful as a form of treatment, although they are usually used. Oral metronidazole is no longer indicated in the treatment of Clostridium difficile-induced diarrhea.

Question 126.    
To reduce the risk of botulism, infants should not consume which of the following?
(a) Unpasteurized milk.
(b) Raw steak.
(c) Honey.
(d) Ice cream.
Answer:
(c) Honey.

Infants under 12 months should not be fed honey due to the risk of infections with Clostridium botulinum. However, most cases of infant botulism are idiopathic.

Question 127.    
Which of the following is most likely to cause foodborne botulism?
(a) Home-canned foods.
(b) Condensed milk.
(c) Tinned tomatoes.
(d) Red wine.
Answer:
(a) Home-canned foods.

The concern is that if home-canned foods are not processed correctly, they can harbor Clostridium botulinum spores. Foods like vegetables, fish, poultry, fruits, pork, and dairy products are most implicated. Non-canned foods have been implicated in outbreaks in restaurants.

Question 128.    
Which of the following is not a complication of measles?
(a) Encephalitis.
(b) Conjunctivitis.
(c) Pneumonia.
(d) Shingles. 
Answer:
(d) Shingles.

Reactivation (shingles) is not a complication of measles infection. It is a complication of chickenpox infection. Complications of measles are encephalitis, conjunctivitis, pneumonia, purpura, sepsis, and hepatitis.

Question 129.    
What type of vaccine is the varicella vaccine?
(a) Toxoid.
(b) Antitoxin.
(c) Immunoglobulin.
(d) Live attenuated.
Answer:
(d) Live attenuated.

The chickenpox vaccine is a live attenuated vaccine. It is derived from a weakened form of the virus. 

Question 130.    
Which of the following factors can lead to an artificially elevated urea level in the blood?
(a) A recent high-protein meal.
(b) Administration of aspirin.
(c) Intravenous hydration.
(d) Alcohol consumption.
Answer:
(a) A recent high-protein meal.

Urea is produced in the liver from the breakdown of proteins. If a person consumes a high- protein meal, there might be a temporary increase in blood urea levels, giving an artificially elevated reading not necessarily associated with renal dysfunction. While some medications and conditions can affect urea levels, of the choices given, only a high-protein meal has this direct relationship.

Question 131.
Which symptom is most commonly associated with peripheral arterial disease (PAD)?
(a) Warm and reddened extremities.
(b) Swelling around the ankles and feet.
(c) Intermittent claudication during exercise.
(d) Pruritus and skin flaking.
Answer:
(c) Intermittent claudication during exercise.

Intermittent claudication refers to pain or cramping in the lower leg that occurs during exercise due to decreased blood flow. It is one of the main symptoms of PAD. This pain typically subsides with rest but returns when the activity is resumed.

Question 132.    
Which of the following is not an expected complication in a patient recently placed on IV opioids for third-degree burns?
(a) Constipation.
(b) Nausea.
(c) Itching.
(d) Respiratory depression.
Answer:
(d) Respiratory depression.

This is not an expected complication because the appropriate dose of opioids is given based on the patient’s current physiology. Respiratory depression is a consequence of overdose and abuse, which are both rare in a controlled setting like a hospital.

Question 133.    
Fluid intake is monitored in a patient with meningitis to reduce the risk of which of the following?
(a) Rebound hypertension.
(b) Pulmonary edema.
(c) Cerebral edema.
(d) Heart failure. 
Answer:
(c) Cerebral edema.

Although fluid overload can increase the risk of pulmonary edema and heart failure, in this patient with an existing cerebral pathology, the risk of cerebral edema is greater.

Question 134. 
A patient who is on chemotherapy for acute lymphoblastic leukemia has anorexia. Which of the following interventions is most appropriate in encouraging this patient to eat?
(a) Pass a nasogastric tube.
(b) Serve spicy foods.
(c) Give IV vitamin B complexes.
(d) Serve small, frequent meals.
Answer:
(d) Serve small, frequent meals.

Small meals can reduce the risk of nausea and vomiting, and serving food frequently can encourage the patient to eat. Passing a nasogastric tube should be a last resort in the face of malnutrition. Spicy foods can worsen the patient’s appetite. IV vitamin B complexes may not stimulate the patient’s appetite. Moreover, vitamin B complexes like folic acid can stimulate the growth of cancer cells.

Question 135.    
Which of the following is a standard goal in managing a patient with raised intracranial pressure?
(a) Blood pressure: 150/100-140/90 mmHg.
(b) ICP: 25-30 mmHg.
(c) PaC02: 26-30 mmHg.
(d) Cerebral perfusion pressure: 50-70 mmHg.
Answer:
(d) Cerebral perfusion pressure: 50-70 mmHg.

In managing raised ICP, cerebral and intracranial perfusion pressure must be monitored. The goal of monitoring is to keep ICP at or below 20 mmHg and cerebral perfusion pressure at 50-70 mmHg. The blood pressure of patients with malignant hypertension should not be crashed. Patients with hypotension are rapidly managed to improve cerebral perfusion. Hypocapnia is not a goal of treatment unless the managing physician decides to commence hyperventilation as a treatment option. 

Question 136.    
Which of the following describes the purpose of misoprostol in a 45-year-old male being managed for erosive ulcers from chronic use of ibuprofen?
(a) Proton pump inhibitor.
(b) Increases pH of the stomach.
(c) Increases mucosal resistance.
(d) Inhibits secretion of somatostatin.
Answer:
(c) Increases mucosal resistance.

Misoprostol is a synthetic prostaglandin analog used to reduce the risk of bleeding in patients with erosive ulcers. Prostaglandin is used in making the protective lining that cushions the stomach mucosa from the effects of hydrochloric acid. Prostaglandin also reduces the secretion of gastrin from the G cells in the antrum of the stomach. Gastrin stimulates the release of gastric acid from the parietal cells.

Question 137.    
Which of the following is unlikely to be elevated on dipstick urinalysis in a 25-year-old female who presents to the ER with clinical features of acute pyelonephritis?
(a) Nitrites.
(b) Red blood cells.
(c) Specific gravity.
(d) Urobilinogen.
Answer:
(d) Urobilinogen.

Urobilinogen is a byproduct of bilirubin formed by the action of bacterial flora in the gut. It is not expected to be increased in acute pyelonephritis unless there is existing liver disease or increased hemolysis. Nitrites are increased in acute pyelonephritis due to bacterial action on nitrates in the urine. Bacteria that release nitrate reductase are E. coli and Klebsiella. Red blood cells can be increased if there is hemolysis. Increased specific gravity means there are increased solutes in the urine due to either increased excretion of these
solutes or dehydration. In acute pyelonephritis, excessive excretion of protein, red blood cells, and white blood cells can increase the urine’s specific gravity.

Question 138.    
A 26-year-old female presents to the ER with a history of lower abdominal pain, purulent vaginal discharge, and fever. On examination, there is positive cervical motion tenderness and strawberry cervix. Which of the following organisms is most implicated?
(a) Gonorrhea.
(b) Chlamydia.
(c) Trichomonas vaginalis.
(d) Candida albicans. 
Answer:
(c) Trichomonas vaginalis.

Strawberry cervix describes an erythematous cervix with a papilliform and punctate appearance. The most implicated organism is Trichomonas vaginalis, which is a flagellate protozoan.

Question 139. 
Which of the following treatment modalities is most suitable for use in managing a 20-year-old male with gonococcal urethritis?
(a) Ceftriaxone + azithromycin.
(b) Ceftriaxone + gentamicin.
(c) Cefuroxime + gentamicin.
(d) Doxycycline + gentamicin.
Answer:
(a) Ceftriaxone + azithromycin.

The recommended dose of ceftriaxone for uncomplicated gonococcal infections is 500 mg IM. Treatment recommendations can change based on local resistance patterns and other factors, so always refer to the latest guidelines or recommendations when choosing treatment. An alternative regimen is a stat dose of 400 mg of oral cefixime with 1 g of oral azithromycin. In patients with allergies to azithromycin, doxycycline is used instead. Patients with allergies to cephalosporins like ceftriaxone are treated with either gentamicin or Gemifloxacin. Monotherapy is not used due to the increasing prevalence of antibiotic resistance.

Question 140.    
A 19-year-old female managed for acute cervicitis secondary to chlamydial infection is about to be discharged. Which of the following is not useful in reducing this patient’s reinfection risk?
(a) Treating all sexual partners.
(b) Contraceptives.
(c) Abstinence.
(d) Completed antibiotic dose.
Answer:
(b) Contraceptives.

Contraceptives can prevent unwanted and unplanned pregnancies. However, they do not protect from sexually transmitted diseases. To reduce the risk of recurrence, all of this patient’s sexual partners must be screened and treated. They should also abstain from all sexual activities until they have completed their treatment regimen (which is about seven days). 

Question 141.    
A patient with acute lymphoblastic leukemia is premedicated with allopurinol before the commencement of chemotherapy. Which of the following best explains the rationale behind using this drug?
(a) Increases excretion of uric acid.
(b) Reduces production of uric acid.
(c) Alkalinizes urine.
(d) Inhibits excretion of calcium.
Answer:
(b) Reduces production of uric acid.

Allopurinol is a xanthine oxidase inhibitor that prevents the conversion of nucleic acid metabolites to uric acid. It is given to patients with a massive tumor load at least two days before the commencement of chemotherapy.

Question 142.    
A patient being managed with cyclophosphamide must be given which of the following as premedication?
(a) Pyridoxine.
(b) Mesna.
(c) Ondansetron.
(d) Prednisolone.
Answer:
(b) Mesna.

Hemorrhagic cystitis is a complication of cyclophosphamide. To reduce the risk of this occurring, all patients must be premedicated with mesna. Mesna is a water-soluble agent that concentrates in the urine and binds with acrolein, a urotoxic metabolite, to form a stable and inactive compound that is readily excreted in the urine.

Question 143.    
Which of the following is not an indication of reverse barrier nursing?
(a) AIDS.
(b) Aplastic anemia.
(c) SCID.
(d) SARS. 
Answer:
(d) SARS.

Reverse barrier nursing is indicated for patients with severe neutropenia and immunosuppression. This form of isolation is done to protect the patient from infection via people and the environment. Principles used in reverse barrier nursing are the provision of a positive pressure isolation room, handwashing, and the use of adequate PPE when visiting the patient.

Question 144. 
Tumor lysis syndrome is likely to be seen in which of the following conditions?
(a) Osteosarcoma.
(b) Non-Hodgkin’s lymphoma.
(c) Cervical carcinoma.
(d) Melanoma.
Answer:
(b) Non-Hodgkin’s lymphoma.

Tumor lysis syndrome is a metabolic syndrome characterized by hyperkalemia, hyperuricemia, hyperphosphatemia, and hypocalcemia. It is caused by the rapid destruction of malignant blood cells and the release of their inflammatory intracellular components into the bloodstream. It occurs mainly in non-Hodgkin’s lymphoma and acute leukemias.

Question 145.    
Which of the following is not used to describe vision loss in amaurosis fugax?
(a) Unilateral.
(b) Temporary.
(c) Painful.
(d) Severe.
Answer:
(c) Painful.

Loss of vision in amaurosis fugax is painless. Other features of vision loss are sudden, temporary, and severe unilateral or bilateral loss.

Question 146.    
A 54-year-old female with amaurosis fugax is being managed in the ER. Which of the following causes is unlikely?
(a) Transient ischemic attack.
(b) Atherosclerosis.
(c) Neovascularization.
(d) Cataract.
Answer:
(d) Cataract.

Causes of amaurosis fugax include embolization from atherosclerotic plaque, endocarditis, atrial myxoma, and fat. Other causes include sudden occlusion of the retinal arteries from transient ischemic attack and bleeding from neovascularization. Cataracts are an unlikely cause of the condition. 

Question 147.    
Which of the following is not a clinical feature of retinal detachment?
(a) Decreased vision.
(b) Eye pain.
(c) Floaters.
(d) Photopsia.
Answer:
(b) Eye pain.

Retinal detachment is a painless eye condition. Symptoms include blurred vision, photopsia, and floaters. Peripheral and central vision may be lost in severe cases. As the detachment worsens, the patient complains of grayness or veiling of the affected visual field.

Question 148.    
A patient who presents to the ER with retinal detachment will require which of the following investigations to confirm their diagnosis?
(a) Tonometry.
(b) Slit lamp test.
(c) Ophthalmoscopy.
(d) Gonioscopy.
Answer:
(c) Ophthalmoscopy.

Retinal detachment is diagnosed via indirect ophthalmoscopy facilitated with dilatation of the pupils. This investigation not only diagnoses the detachment but also differentiates the different types of detachment. Tonometry is used to determine intraocular pressure and diagnose glaucoma; a slit lamp test is used to diagnose ulcers and abrasions of the cornea; gonioscopy is used to assess the iridocorneal angle and diagnose angle-closure glaucoma.

Question 149.    
Which of the following clinical manifestations is most consistent with lamotrigine toxicity?
(a) Respiratory depression.
(b) Dry mouth and urinary retention.
(c) Nystagmus and ataxia.
(d) Profound hypotension. 
Answer:
(c) Nystagmus and ataxia.

Lamotrigine tonicity can lead to various neurologic consequences. The most common symptoms of lamotrigine overdose include nystagmus (involuntary eye movement) and ataxia (uncoordinated movement). The other options might be seen with various drug overdoses, but they are not as specifically tied to lamotrigine toxicity as nystagmus and ataxia.

Question 150.    
Which of the following is not a feature of globe laceration?
(a) Visible lacerations of the sclera or cornea.
(b) Positive red reflex.
(c) Shallowness of the anterior chamber.
(d) Raised intraocular pressure.
Answer:
(b) Positive red reflex.

In globe laceration, the red reflex may be absent or abnormal. Other features of globe laceration are visible lacerations of the sclera or cornea, shallowness of the anterior „ chamber, raised intraocular pressure, nausea, and vomiting.

Question 151.    
Which of the following is not a feature of opioid overdose?
(a) Miosis.
(b) Bradycardia.
(c) Hypotension.
(d) Diaphoresis.
Answer:
(d) Diaphoresis.

Diaphoresis is a sign of opioid withdrawal. Features of opioid overdose are hypotension, bradycardia, respiratory depression, miosis, urinary retention, coma, and death.

Question 152.    
A woman who presents to the ER with Valium poisoning may be managed with which of the following?
(a) Naltrexote.
(b) Flumazenil.
(c) Gabapentin.
(d) Disulfiram.
Answer:
(b) Flumazenil.

Flumazenil is a benzodiazepine receptor blocker. It is given in acute intoxication to reverse sedation and respiratory depression.

Question 153.    
A man who presents to the ER with nicotine withdrawal symptoms is being managed with bupropion. Which of the following best describes the function of this drug in this patient?
(a) Anxiolytic.
(b) Mood stabilizer.
(c) Synthetic nicotine.
(d) Dopamine reuptake inhibitor.
Answer:
(d) Dopamine reuptake inhibitor.

Bupropion is a norepinephrine and dopamine reuptake inhibitor. It increases circulating levels of norepinephrine and dopamine, improves mood, and helps patients cope with symptoms of nicotine withdrawal. 

Question 154.    
Which of the following is not a feature of amphetamine intoxication?
(a) Psychosis.
(b) Delirium.
(c) Hyperthermia.
(d) Increased appetite.
Answer:
(d) Increased appetite.

Features of amphetamine intoxication are delirium, toxic psychosis, tachycardia, nausea, vomiting, hyperthermia, seizures, stroke, arrhythmias, and hypertension. In severe cases, rhabdomyolysis and acute kidney injury can occur. Increased appetite is not a feature of the condition.

Question 155.    
Which of the following is the most common cause of Clostridium difficile-induced gastroenteritis?
(a) Contaminated water.
(b) Antibiotics.    
(c) Raw seafood.
(d) Raw steak. 
Answer:
(b) Antibiotics.

Clostridium difficile is an anaerobic bacteria found in the intestine. Infections occur when there is an overgrowth. This overgrowth is usually triggered by an alteration in the normal flora of the gut. The most implicated antibiotics are cephalosporins (especially third-generation), clindamycin, fluoroquinolones, and penicillins.

Question 156.    
A nurse is about to hand over a specimen to the police officer in a chain of custody. They are expected to do all of the following except:
(a) Request identification from the police officer.
(b) Examine the specimen in front of the police officer.
(c) Document and sign.
(d) Have the police officer document and sign.
Answer:
(b) Examine the specimen in front of the police officer.

This is not appropriate because specimens are stored in tamperproof containers and packaged as soon as they are collected. In order to ensure the specimen is given to the designated individual, the nurse should ask for identification. Both parties must document and sign off as the specimen is handed over.

Question 157.    
A 25-year-old female patient comes to the ER and reports a new rash that appeared after she started lamotrigine therapy two weeks ago. This is a potential sign of:
(a) Drug tolerance.
(b) Stevens-Johnson syndrome.
(c) Serotonin syndrome.
(d) Allergic rhinitis.
Answer:
(b) Stevens-Johnson syndrome.

Stevens-Johnson syndrome (SJS) is a rare, serious skin reaction that can be caused by certain medications like lamotrigine. It typically begins with flu-like symptoms and then eventually causes blisters. Recognizing these early symptoms and discontinuing the medication is crucial because SJS can be life-threatening.    

Question 158.    
To assess the severity of pain in an adult male with intact cognition, nurses should use any of the following methods except:
(a) Verbal scale.
(b) Visual analog scale.
(c) Functional pain scale.
(d) Graphic scale.
Answer:
(d) Graphic scale.

The graphic scale, which often includes images of faces that range from grimacing to smiling or fruits of varying sizes, is typically only used for children or adults with impaired cognition or limited literacy.

Question 159.    
A five-year-old female with febrile seizures presents to the ER. Which of the following interventions should be prioritized?
(a) Administering IV acetaminophen.
(b) Administering IV diazepam.
(c) Placing the child in the left lateral position.
(d) Tepid sponging.
Answer:
(c) Placing the child in the left lateral position.

This intervention should be prioritized because it reduces the risk of aspiration. After the child is positioned appropriately, IV benzodiazepines and acetaminophen are administered. Tepid sponging is contraindicated.

Question 160.    
Which of the following statements is false when counseling the parents of a four-month-old male who presents to the ER with bronchopneumonia?
(a) Handwashing reduces the risk of infections.
(b) Breastfeeding should only be continued until the baby is five months old.
(c) The baby should be fully immunized.
(d) Iron-rich formula should be commenced when the baby is six months old. 
Answer:
(c) Placing the child in the left lateral position.

This intervention should be prioritized because it reduces the risk of aspiration. After the child is positioned appropriately, IV benzodiazepines and acetaminophen are administered. Tepid sponging is contraindicated.

Question 161.    
A 45-year-old male patient arrives at the hospital complaining that his right ear has been “ringing” for the past three days. He describes it as a continuous, high-pitched sound. This condition is best described as:
(a) Vertigo.
(b) Otitis media.
(c) Tinnitus.
(d) Meniere’s disease.
Answer:
(c) Tinnitus.

Tinnitus is described as the perception of noise or ringing in the ears. It is not a condition but a symptom of an underlying condition, such as age-related hearing loss, ear injury, or a circulatory system disorder. The described symptoms align with tinnitus.

Question 162.    
An elderly patient reports experiencing persistent tinnitus. Which of the following medications, if found in the patient’s history, could be a potential cause?
(a) Acetaminophen.
(b) Aspirin.
(c) Cetirizine.
(d) Metformin.
Answer:
(b) Aspirin.

Certain medications can cause or worsen tinnitus. Aspirin is one medication known to have tinnitus as a side effect (especially in larger doses).

Question 163.    
You are monitoring a 55-year-old female who was admitted with STEMI and is now undergoing heparin therapy. Which of the following medications is required during heparin therapy?
(a) Vitamin K.
(b) Protamine sulfate.
(c) Aminocaproic acid.
(d) Epinephrine.
Answer:
(b) Protamine sulfate.

Protamine sulfate is a reversible antagonist of heparin. It is used for heparin overdose. It is also used to counter the effects of heparin, especially during surgery. Protamine is given via IV as 1.0 to 1.5 mg for every too IU of circulating heparin. It is important to monitor PTT during the commencement of protamine sulfate.

Question 164.    
A 67-year-old male who presents to the ER with crushing chest pain, diaphoresis,-and shortness of breath is given oral aspirin as part of supportive care. Which of the following best describes aspirin’s mechanism of action in this scenario?
(a) Dilates peripheral blood vessels.
(b) Lysis of existing blood clot.
(c) Inhibits platelet aggregation.
(d) Inhibits vitamin K-dependent clotting factors.
Answer:
(c) Inhibits platelet aggregation.

Aspirin is a potent NSAID used to reduce mortality risk in patients experiencing a myocardial infarction. Low-dose aspirin inhibits the production of thromboxane A2 in platelets. This effect reduces platelet aggregation and the formation of a platelet plug. About 325 mg of oral aspirin is given for immediate antiplatelet action. Patients are advised to chew or swallow the drug.

Question 165.    
A 32-year-old primigravida patient is in the active phase of labor. The physician has ordered an oxytocin infusion to augment labor. Which of the following is the most appropriate initial rate for the oxytocin infusion?
(a) 0.5-2 mU/min.
(b) 20-40 mU/min.
(c) 6-12 mU/min.
(d) 50-60 mU/min. 
Answer:
(a) 0.5-2 mU/min.

For the augmentation or induction of labor in a primigravida, the recommended starting dose for oxytocin infusion is typically 0.5-2 mU/min (milliunits per minute). The dose can be increased by 1-2 mU/min every 30 to 60 minutes until a satisfactory contraction pattern is established.

Question 166.    
A 35-year-old male who presents to the ER with testicular torsion is undergoing an urgent manual detorsion of the affected testes. In this procedure, the testis will be rotated in which direction?
(a) Inward.
(b) Outward.
(c) Upward.
(d) Downward.
Answer:
(b) Outward.

The testes rotate inward during torsion. Consequently, the testes are rotated outward during manual detorsion. Success rates are variable, and more than one rotation may be required. If detorsion is unsuccessful, immediate surgery is required.

Question 167.    
A 35-year-old man arrives at the hospital. He reports 12 hours of suprapubic pain and swelling due to an inability to urinate. Upon examination, the nurse finds a significantly distended bladder and is unable to insert size-18 and size-22 catheters. What should be the immediate intervention in this scenario?
(a) IV analgesia.
(b) Cystostomy.
(c) Pass a size 16 catheter.
(d) Infiltrate the bladder with lidocaine.
Answer:
(b) Cystostomy.

An urgent suprapubic cystostomy must be immediately performed by qualified personnel (most likely a surgeon) to avoid the risk of bladder rupture. This patient most likely has a urethral stricture as the cause of his urinary retention. 

Question 168.    
Which of the following measures is most important in preventing a UTI in a 45-year-old male with a urethral catheter for acute urinary retention secondary to fecal impaction?
(a) Use of a closed drainage system.
(b) Change of catheter every two weeks.
(c) Use of a silicone catheter.
(d) Prophylactic antibiotics.
Answer:
(a) Use of a closed drainage system.

A closed drainage system (where the urethral catheter is connected to a urine bag) is the most important measure in preventing a UTI. Prophylactic antibiotics, the use of a silicone catheter, and frequent changing of the catheter are not useful in this patient because his catheterization is short-term management until the fecal impaction is treated.

Question 169.    
A 56-year-old male with chronic urinary retention presented to the ER with difficulty passing urine for six hours. He was immediately catheterized and produced large quantities of diluted urine. This patient must be monitored for which of the following?
(a) Hematuria.
(b) Vasovagal response.
(c) Postobstructive diuresis.
(d) Acute kidney injury.
Answer:
(c) Postobstructive diuresis.

Postobstructive diuresis is polyuria (urine excretion of at least 200 cc for at least two hours) after treatment of urinary retention. It also includes urine excretion of more than 3,000 cc in 24 hours. Causes include poor concentration effect of the kidneys, decreased reabsorption of sodium ions, urea retention, and accumulation of atrial natriuretic peptide.

Question 170.    
Which of the following is an indication of a three-way catheter?
(a) Bladder cancer.
(b) Benign prostatic hyperplasia.
(c) Prostatitis.
(d) Urethral stricture.    
Answer:
(a) Bladder cancer.

Bladder cancer is often characterized by painless hematuria and subsequent urinary retention from blood clots. A three-way catheter is needed for urine excretion and bladder irrigation to dislodge blood clots. A three-way catheter has three lumens: one for inflating the balloon, one for urine drainage, and one for irrigation.

Question 171.    
While administering an oxytocin infusion for labor induction, the nurse notes tachysystole in the patient. What is the most appropriate nursing action?
(a) Increase the oxytocin infusion rate to hasten delivery.
(b) Discontinue the oxytocin infusion and notify the physician.
(c) Administer a bolus of intravenous fluids.
(d) Reposition the patient to her left side.
Answer:
(b) Discontinue the oxytocin infusion and notify the physician.

Tachysystole refers to more than five contractions in 10 minutes averaged over a 30-minute window. When tachysystole is noted, especially when oxytocin is being administered, the infusion should be discontinued immediately, and the physician should be notified. Tachysystole can compromise fetal oxygenation, so it is important to act immediately.

Question 172.    
A 29-year-old female presents to the emergency department with complaints of itching and burning in her genital region. She mentions recently taking antibiotics for a sinus infection. On examination, a white, cottage cheese-like discharge is noted. The nurse recognizes that the most likely causative agent for this presentation is:
(a) Trichomoniasis.
(b) Chlamydia.
(c) Candida.
(d) Gonorrhea.
Answer:
(c) Candida.

Candida is a type of yeast that can cause fungal infections. Vaginal candidiasis, commonly referred to as a yeast infection, can present with symptoms such as itching, burning, and a white, cottage cheese-like discharge. Antibiotics can disrupt the normal flora balance, leading to an overgrowth of Candida in the vagina.

Question 173.    
A 76-year-old male arrives at the ER with an altered mental status and a recent history of poor fluid intake. His serum sodium level is found to be 158 mEq/L. Which of the following clinical manifestations is most commonly associated with hypernatremia?
(a) Muscle cramps.
(b) Bradycardia.
(c) Seizures.
(d) Paresthesias.
Answer:
(c) Seizures.

Hypernatremia, or elevated serum sodium levels, can lead to cellular dehydration. The central nervous system is particularly sensitive to changes in serum osmolality, which can result in symptoms ranging from lethargy and irritability to seizures and coma in severe cases. 

Question 174.    
In managing a patient with severe hypernatremia, a rapid correction of sodium levels can lead to which of the following complications?
(a) Pulmonary edema.
(b) Central pontine myelinolysis.
(c) Hyperkalemia.
(d) Metabolic alkalosis.
Answer:
(b) Central pontine myelinolysis.

A rapid correction of sodium levels can lead to a serious condition called central pontine myelinolysis (CPM), also known as osmotic demyelination syndrome. CPM is a neurological disorder characterized by the loss of myelin in the brainstem, which can result in severe and potentially permanent neurological complications.

Question 175.    
The anal wink reflex is checked on a patient with a potential spinal cord injury. The test involves:
(a) Inserting a gloved finger into the anus and checking for a contraction of the anal sphincter.
(b) Observing spontaneous contractions of the anus in response to sudden loud noises.
(c) Stroking the perianal skin and observing for a contraction of the anal sphincter.
(d) Applying cold stimuli to the anus and observing for anal shivering.
Answer:
(c) Stroking the perianal skin and observing for a contraction of the anal sphincter. 

The anal wink reflex is a simple clinical test that involves stroking or pinching the perianal skin to elicit a contraction of the anal sphincter. It is used to evaluate the integrity of the sacral spinal cord segments S2-S4. An absent or abnormal reflex might indicate an injury or lesion in the corresponding sacral spinal cord segments or nerve roots.

Practice Tests:

Book an appointment